You are on page 1of 104

1.

Consider the following statements:

1) Mount Aconcagua is located in Africa.

2) The Atacama Desert is located in Peru.

Which of the statements given above is/are correct?

A. 1 only
B. 2 only
C. Both 1 and 2
D. Neither 1 nor 2

Answer ||| D

Solution |||

Aconcagua is the highest mountain in South American continent. It is located on the


border of Argentina and Chile.

The driest place on earth is the Atacama Desert in Chile. It is a plateau that occupies a 600
mile

strip of the Pacific coast on the western side of Andes Mountains.

2.Assertion (A): A sinkhole represents an opening more or less circular at the top and funnel-
shaped towards the bottom.

Reason (R): Sinkhole usually has an opening through which cave streams are discharged.

Codes:

A. Both A & R are individually true, and R is the correct explanation of A


B. Both A & R are individually true, but R is not the correct explanation of A
C. A is true, but R is false
D. A is false, but R is true

Answer ||| C

Solution |||

Sinkholes are small to the medium-sized rounded to sub-rounded shallow depressions known
as swallow holes forms on the surface of rocks such as limestone by the action of the
solution. A sinkhole represents an opening more or less circular at the top and funnel-shaped
towards the bottom. When a sinkhole is formed solely through the process of solution, it is
known as a solution sink. Some sinkhole initiated its formation through the solution process
but with time it gets collapse due to the presence of some caves/hollow beneath it and
becomes a bigger sinkhole. These types are known as the collapse sinks. The phenomenon
Doline is sometimes used to refer as collapse sinks. Solution sinks are more common when
compared to collapse sinks. When many sink holes join together to form a valley of sinks,
they are known as valley sinks or Uvalas. Lapies are an irregular grooves and ridges formed
when most of the surfaces of limestone are eaten by the solution process.

Caves are in the areas where there are alternative beds of rocks (i.e. in a non-soluble form)
with the limestone or dolomite present in between or in areas where the limestone are quite
dense, massive and occurring as the thick beds, cave formation is prominent. Caves usually
have an opening through which the cave streams are discharged. Caves possess an opening at
both the ends are known as tunnels.

3.Consider the following statements:

1) When the Meteors reaches the earth, it is known as Meteorite.

2) Meteor shower are the series of Meteorites, when they encountered at once.

Select the correct answer from the codes:

A. 1 only
B. 2 only
C. Both 1 and 2
D. Neither 1 nor 2

Answer ||| D

Solution |||

The Meteors are the bits of rock & ice that are ejected from the comets as they get to
manoeuvre around their orbits around the sun. The Orionids meteors get to emerge from the
comet 1P/Halley.

The Meteor showers are witnessed when the Earth passes through the trail of debris left by a
comet or by an asteroid.

When the meteor reaches the Earth, it is known as a meteorite and the series of meteorites
when they encountered at once, is described as a meteor shower.

As it gets fall towards the Earth, the resistance by it makes the space rock quite extremely hot
and as the meteorite get passes through the atmosphere, it leaves behind the streak of hot
glowing gas that get visible to the observers and not to the rock itself.

4.Which among the following is incorrect with respect to the Humidity -


A. Absolute humidity is expressed in gm/kg unit.
B. Specific humidity is not get affected by a change in pressure or temperature.
C. Relative humidity is measured by Hygrometer.
D. None of the above

Answer ||| A

Solution |||

Humidity:

It is defined as the amount of water vapour present in air.

At any specific temperature, amount of the water vapour that can be held by the air has a
definite limit called as saturation point.

Air at the saturation point is called as saturated air.

Temperature at which the saturation occurs is called as Dew point.

Capacity of air to absorb the water vapour increases with the increase in temperature.

Absolute humidity:

• It is the weight of actual amount of water vapour present in unit volume of air.

• It is expressed in gm / m3.

Specific humidity:

• It is the weight of water vapour present in per unit weight of air.

• It is not affected by the change in pressure/temperature.

• It is expressed in gm/kg.

Relative humidity:

• It is the ratio of water vapour in air at a particular temperature to the total amount of water
vapour required to saturate the same air at the same temperature.

• Relative humidity increases with the increase in water vapour in air and decreases with
increase in temperature.

• Relative humidity is measured with the help of Hygrometer.

• Expressed in %
5.Sandstone is a sedimentary rock, when get converted into the metamorphic rock it becomes

A. Marble
B. Graphite
C. Schist
D. Quartzite

Answer ||| D

Solution |||

Metamorphic rocks:

• These rocks formed when the original structure of the igneous and sedimentary rocks
partially or wholly gets change under the action of heat and pressure.

• They contain no fossils.

• There is no stratification in these rocks.

• For example, Clay converted into Slate.

Sedimentary to Metamorphic

• Limestone → Marble

• Sandstone → Quartzite

• Shale → Schist

• Coal → Graphite

• Bituminous coal → Anthracite coal

Igneous to Metamorphic

• Granite → Gneiss

• Mica → Schist

• Gabro → Serpentine

6.Which among the following is/are the technique of soil erosion prevention?

1) Contour trenching

2) Cover crops
3) No-till farming

4) Strip cropping

Select the correct code from given below:

A. 1 and 2 only
B. 3 and 4 only
C. 1, 2 and 3
D. All are correct

Answer ||| D

Solution |||

Contour barriers or Contour trenching

• Contour strips which get intercept downslope flowing water & soil particles.

• These barriers result in slow down of the water movement and reduce its erosive force.

• They even filter out and trap several of the suspended soil particles, keeping them far from
being washed out of the field.

Cover crops or Crop rotations

• Cover crops like legumes, radishes white turnip and other species are rotated with the cash
crops to blanket the soil year-round.

• They act as a green manure that replenishes the nitrogen and other critical nutrients.

• They also help in suppressing the weeds and increase soil fertility.

No-till farming

• They are also known as zero till age/direct drilling.

• It is a way of growing crops or pasture from one year to the other without disturbing the soil
through tillage.

• It increases the amount of water that get infiltrates into the soil.

• They also increase the organic matter retention & cycling of nutrients in the soil.

• It keeps soils anchored in the place rather than having bare ground exposed to the wind and
water.

Strip Cropping
• Crops are grown in the alternate strips of the land to check the impact of the winds.

• It is used when the slope is too steep or when there is no other alternative method of
preventing the soil erosion.

• Contour strip cropping: Cultivation of the soil protecting crops in strips alternating with the
erosion permitting crops.

• Field strip cropping: Plants are cultivated in the parallel strips across the slopes.

7.Which among the following archipelago is also known as Manus Islands?

A. Chonos Archipelago
B. Aguni Islands
C. Bonin Islands
D. Admiralty Islands

Answer ||| D

Solution |||

The Admiralty Islands are the archipelago group of 18 islands in total in the Bismarck
Archipelago, to the north of New Guinea located in the South Pacific Ocean. These are even
sometimes known as the Manus Islands, after the largest island. Several of the Admiralty
Islands are the atolls and uninhabited.

The larger islands in the centre of the group are the Manus Island & Los Negros Island. The
other larger islands are the Tong Island, Rambutyo Island, Pak Island, Lou Island, and Baluan
Island to the east side, Mbuke Island to the south direction and Bipi Island to the west side of
Manus Island.

The temperature of the Admiralty Islands (Manus islands) varies little all over the year,
reaching daily highs of 30 to 32 °C and 20 to 24 °C at night. Average annual rainfall is
around 3,382 mm and is somewhat seasonal, with June to August being the wettest months.

Because of its isolated location, the rain forests of the Manus Islands are home to rare &
endemic species of birds, bats and also other animals, and are considered the separate
ecoregion, the Manus Islands lowland rain forests. The majority forests on the Manus islands
still remain, but few of the smaller islands have been cleared for the coconut farming. The
typical tree species are many Calophyllum & Sararanga species.

8.Which among the following statements is correct regarding the Cloud types:

A. Cirrus clouds are generally formed at the height of 4,000 m to 7,000 m.


B. Stratus clouds are vertical.
C. Altostratus & Altocumulus clouds come under the ‘Middle clouds’ category.
D. Nimbus clouds look like a cotton wool.

Answer ||| C

Solution |||

According to their altitude, density, stretch and transparency or opaqueness clouds are
categorised into 4 types which are given below:

Cirrus Clouds:

* Cirrus clouds get formed at high altitudes of 8,000 to 12,000m.

* They are the detached thin clouds.

* These clouds have a feathery appearance.

* These clouds are always white in colour.

Cumulus Clouds:

* Cumulus clouds are usually formed at the height of 4,000 m to 7,000 m.

* These clouds look like a cotton wool.

* They exist in the patches and can be view dispersed here & there.

* They possess a flat base.

Stratus Clouds:

* These clouds are horizontal.

* They are stratified/layered clouds cover major portions of the sky.

* These clouds are generally formed due to the mixing of air masses with several
temperatures or due to the loss of heat.

* The presence of these clouds means chilly, overcast day.

Nimbus Clouds:

* Nimbus clouds are generally formed at a lower altitudes.

* The colour of these clouds is usually black/dark grey.

* These clouds block the sunlight.


* These clouds generally cause heavy rainfall & thunderstorms.

9.The number of people per unit area of arable land is termed as

A. Agricultural density
B. Arithmetic density
C. Physiological density
D. Economic density

Answer ||| C

Solution |||

The total number of people per unit area of arable land is known as Physiological density.

A higher the physiological density suggests that the available agricultural land is being used
by more and may reach its output limit sooner than a country that has a lower physiological
density.

Population density is often measured in three different ways. They are Arithmetic density,
physiological density, and Agricultural density.

* The first method used to calculate population density is the Arithmetic density, which is the
total number of people in any given particular area as compared to one square unit of land.
The total number of people is splited by, for instance, one kilometre, to determine the average
density on that acre.
* Agricultural density calculates the number of farmers, specifically on every unit of
farmland. This signifies people who work the earth on the specific plots of land that are used
for this. It can still measure by kilometre, but it only measures kilometres of farmable land.

* Physiological density calculates the total number of people and divides them among the
total amount of farmable land. So the proportion of land to be used is much smaller than for
the arithmetic density, but the amount of people is much larger than what is used in the
agricultural density.

10.The term soil impoverishment relates to which one of the following?

A. Soil erosion
B. Soil deposition
C. Soil getting very deficient in plant nutrients
D. Soil getting enriched with plant nutrients

Answer ||| A

Solution |||

Soil impoverishment actually means a number of conditions that can lead to degradation in
the quality of soil, or degradation in its richness. It relates to soil erosion. This might be
caused due to reckless cutting of trees, leading to deforestation of trees. We know that trees
help to bind the soil to the earth, and hence once these are uprooted, the richness gets
diminished. Also, land should not be used carelessly for other activities that can harm its
fertility.

Soil impoverishment is based on the innate ability of prairie plants to tolerate and flourish in
soils with low levels of nitrogen. Soil impoverishment, or reverse fertilization, involves the
removal of nutrients from the soil. It is most often done by introducing large amounts of
organic Matter to the soil.

11.In the Hadley cell thermal circulation, air rises up and finally descends at

A. Intertropical convergence zone


B. Doldrums
C. Subtropical high-pressure cells
D. Equatorial troughs

Answer ||| C

Solution |||
Hadley cell is an atmospheric circulation in which the air rises near the equator due to intense
heating and then it flows poleward and due to coriolis force it descends at the subtropical
high pressure belt. Then again the descended air returns equatorward and forms the Hadley
cell. This also leads to the creation of trade winds.

12.Which of the following statement is not correct about thunderstorms?

A. Thunderstorms are caused by intense convection on moist hot days.


B. A thunderstorm is a well-grown cumulonimbus cloud producing thunder and lightning.
C. A thunderstorm is characterised by intense updraft of rising warm air, which causes the
clouds to grow bigger and rise to greater height.
D. Thunderstorms are frequent in Polar Regions and at latitudes higher than 50° N and 50° S.

Answer ||| D

Solution |||

A thunderstorm is caused by the intense convention on hot moist days and is characterised by
the intense updraft of rising warm air, which causes the clouds to grow bigger and rise to a
greater height.

* Thunderstorms are rare in Polar Regions and are less frequent at latitudes higher than 50°
N and 50° S.

* They are most frequent in the mid-latitudes, where warm, moist air from tropical latitudes
collides with cooler air from polar latitudes.

13.Which of the following Indian city lies nearest to the latitude passing through the Capital
city of United Arab Emirates, Abu Dhabi?

A. Hyderabad

B. Bhopal

C. Mumbai

D. Surat

Answer ||| B

Solution |||
14.Which of the following statements is incorrect about El Nino?

A. The warm water of the central Pacific Ocean slowly drifts towards the South American
coast and replaces the cool Peruvian current. Such appearance of warm water off the coast of
Peru is known as the El Nino.
B. The El Nino event is closely associated with the pressure changes in the Central Pacific
and Australia.
C. The combined phenomenon of southern oscillation and El Nino is known as ENSO.
D. La Nina is the opposite of El Nino as is characterised by ‘warming’ of the ocean water in
parts of the Pacific Ocean.

Answer ||| D

Solution |||

EL Nino is characterised by warming of the Pacific Ocean and La Nina, which is considered
as the opposite of El Nino as is characterised by ‘cooling’ of the ocean water in parts of the
Pacific Ocean.

15.Which one of the following ocean currents is a cold current?


A. South Atlantic Drift
B. Mozambique Current
C. East Australian Current
D. Caribbean Current

Answer ||| A

Solution |||

Warm ocean currents flow away from the equatorial region on the western side of ocean
basins whereas Cold currents flow toward the equator on the eastern side of ocean basins.

Cold Currents -

1. California Current

2. Humboldt Current

3. Labrador Current

4. Canaries Current

5. Benguela Current

6. Falkland Current

7. West Australian Current

8. Okhotsk Current

Warm Currents -

1. North Pacific Drift

2. North Equatorial Current


3. Equatorial Countercurrent

4. South Equatorial Current

5. West Wind Drift

6. Gulf Stream

7. North Atlantic Drift

8. North Equatorial Current

9. Equatorial Countercurrent

10. South Equatorial Current

11. Brazil Current

12. West Wind Drift

13. Monsoon Current

14. Equatorial Countercurrent

15. South Equatorial Current

16. Mozambique Current

17. West Wind Drift

18. Japan Current

19. North Equatorial Current

20. Equatorial Countercurrent

21. South Equatorial Current

22. East Australian Current

16.According to the Koppen climatic classification, the letter code Cfa denotes

A. Tropical wet climate.


B. Humid subtropical climate.
C. Tundra climate.
D. Tropical wet and dry climate.
Answer ||| B

Solution |||

The Köppen Climate Classification divides the Earth's climate into five main climate groups:

• A (tropical)

• B (dry)

• C (temperate)

• D (continental)

• E (polar)

The second letter indicates the seasonal precipitation type, while the third letter indicates the
level of heat.

The letter code "Cfa" denotes the humid subtropical climate. In these places, the summers are
hot and humid with frequent thunderstorms. The winters are comparatively mild and
precipitation during this period occurs due to mid-latitude cyclones, usually in areas in and
around the southeastern USA.

17.Which one of the following rivers is NOT a tributary of river Cauvery?

A. Hemavati
B. Arkavati
C. Indravati
D. Amravati

Answer ||| C

Solution |||

The river Indravati is a tributary of the river Godavari river not of Cauvery, which is located
in central India. The river gets rises in the Eastern Ghats in Odisha and flows westwards to
join the Godavari. There is a famous wildlife sanctuary named Indravati National Park on its
bank.

Tributaries of river Cauvery are: Amaravati, Arkavati, Bhavani, Chinnar, Hemavati,


Honnuhole, Kabini, Kannika, Kollidam, Lakshmana Tirtha, Lokapavani, Noyyal, Pambar,
Shimsha, Sujyothi

Left Bank tributaries: the Harangi, the Hemavati, the Shimsha and the Arkavati.
Right Bank tributaries: the Lakshmantirtha, the Kabbani, the Suvarnavati, the Bhavani, the
Noyil and the Amaravati joins from the right.

Riparian States and UTs

Karnataka, Kerala, Pondicherry, Tamil Nadu

Cauvery River Basin

The Kaveri basin is estimated to 72,000 km2 with many tributaries including the Shimsha,
the Hemavati, the Arkavati, Honnuhole, Lakshmana Tirtha, Kabini, Bhavani River, the
Lokapavani, the Noyyal and the Amaravati River.

18.Which one of the following is a local wind that blows out from Siberia?

A. Bora
B. Purga
C. Mistral
D. Blizzard

Answer ||| B

Solution |||

Purga is a violent cold wind with drifting snow. It blows out from Siberia.

• Purga is a violent cold wind which blows in the Russian tundra region.

• Bora is a cold and dry wind which blow outwards from Hungary to the north of Italy in the
upper Adriatic Sea.

• Mistral is a cold wind blows in the Rhone valley of France.

• Blizzard is wind or storm which contains large amounts of snow and blows in the Tundra
region.

19.Which of the following are essential prerequisites for the establishment of a thermal power
station?

1) Availability of fossil fuels

2) Proximity to a river, lake or sea

3) Good transport network


4) Proximity to an urban centre

Select the correct answer using the code given below:

A. 1, 2 and 3
B. 2 and 4
C. 2 and 3 only
D. 1 and 3 only

Answer ||| A

Solution |||

A thermal power station is the power station in which heat energy is get converted to electric
power. In most, a steam-driven turbine turns heat into mechanical power as an intermediate to
electrical power. Water is heated, which turns into steam and drives a steam turbine which
operates an electrical generator. After it let passes through the turbine, the steam gets
condensed in a condenser and recycled to where it was heated. This is called as a Rankine
cycle.

The power plant is used for generating power which signifies that it will consume huge
quantities of the fuel. In India, majority of thermal power plants use coal as a fuel.

There should be ample space for the storage of coal, disposal of ash, the building of the
power plant, and a residential colony of workers, proximity to the river, lake etc., markets,
good transport and communication and so forth. Therefore, the power plant site needs to have
a good amount of the land, and this land must have the good bearing capacity in order to
survive the static and dynamic loads during the operation of the plant.

20.The gently sloping accumulation of coarse alluvium deposits by a braided stream is known
as

A. Sand bar
B. Alluvial fan
C. Bajada
D. Diaras

Answer ||| A

Solution |||

The gently sloping accumulation of coarse alluvial deposits by the braided stream is known
as a sand bar.
• A sandbar is a portion of sand, gravel or fine sediment that sits above the water. It may be
get connected to the shoreline, or it may be offshore. It is usually narrow and straight. A
sandbar is also called as a shoal or a sandbank. A large sandbar is also called a barrier island.

• Sandbars begin forming underwater. As the waves break, this attracts material from the
shoreline, migrating even further into the ocean. During the time of heavy storms, large
waves can build sandbars far from the shore, until they rise above the water's surface.

Sandbar Landforms Have 3 Main Characteristics:

• Made of sand, silt or gravel

• Formed by the wave action and currents

• Generally forms the straight line

Famous Sandbars (or Barrier Islands):

• Miami Beach, Florida, USA

• Bolivar Island, Texas, USA

21.Which one of the following Forest Acts divided forests of India into reserved, protected
and village forests?

A. Forest Act 1864


B. Forest Act 1865
C. Forest Act 1866
D. Forest Act 1878

Answer ||| D

Solution |||

The Indian Forest Act 1878 divided the forest into:

ÂReserved forest: it is completely controlled by the government

ÂProtected forest: it is partially controlled by the government

ÂVillage forest: controlled by village people

It was amended by the Indian Forest act 1927.


22.The north-west part of the Indian peninsula was converted into a great center of
vulcanicity in

A. the end of the Cretaceous period


B. the mid of Cretaceous period
C. the start of the Cretaceous period
D. the whole of the Cretaceous period

Answer ||| D

Solution |||

The geography of the rest of India is such that, during the Cretaceous period, the north-west
area of the Peninsula was turned into a great center of vulcanicity of a type which has not a
single parallel among the volcanic phenomena of the present world.

23.Consider the following statements:

1) Wheat is sown in mid-October to mid-November and harvested in March.

2) Wheat grows well in the cool, moist climate and ripens in a warm, dry climate.

Which among the following given statements is/are correct about climatic conditions for
growing Wheat?

A. Only 1
B. Only 2
C. Both 1 and 2
D. Neither 1 nor 2

Answer ||| C

Solution |||

Wheat is a Rabi crop. It is sown in mid-October to mid-November and harvested in March. It


grows well in cool, moist climate & ripens in a warm, dry climate. The cool winters & the hot
summers are conducive to a good crop. A cloudless sky having bright sunshine during
ripening & harvesting periods will make better quality wheat.

There are two major types of crops:

* Kharif Crops or Monsoon Crops are domesticated plants cultivated and harvested during
the rainy (monsoon) season in South Asia, which lasts between April and October depending
on the area. Main Kharif crops are millet and rice. Kharif crops are generally sown with the
beginning of the first rains in July, during the south-west monsoon season.
* Rabi crops or Rabi Harvest are agricultural crops sown in winter and harvested in the spring
in South Asia. The term is obtained from the Arabic word for “spring”, which is used in the
Indian subcontinent, where it is the spring harvest (also known as the “winter crop”). The rabi
crops are sown around mid-November after the monsoon rains are over, and harvesting
begins in April/May. The crops are grown either with rainwater that has percolated into the
ground or with irrigation.

24.Which among the following given statements is/are correct?

1) The parallels of latitude are of equal length.

2) The equator & the Prime Meridian are the two largest circles which can be drawn on the
earth.

3) Indian Standard Time & the local time are the same.

4) We count 180 Degree Meridian of longitude as both East and West.

Code:

A. Only 1
B. Only 2
C. Only 3
D. Only 4

Answer ||| D

Solution |||

The parallels of latitude are not of equal length but of different lengths. The Equator is the
largest possible circle which can be drawn around the earth, while Prime Meridian is an only
semi-circle of value ‘00’ longitudes.

Indian Standard Time is the common time for whole India, whereas local time is calculated
when the sun is overhead at that particular place.

We count 180 Degree Meridian of longitude as both east and west.

Latitude of a place is considered as the angular distance of the place from the equator. All
latitudes, also known as Parallels, are concentric circles. The Equator is the largest latitude of
the earth, splitting the earth into two equal halves.

Longitude of a place is considered as the angular distance of the place from the Prime
Meridian. Longitudes, also consider as Meridians, play an important role in measuring the
time zone of a location. Zero degrees longitude is Greenwich in the UK, and 180 degree is
the International Date Line.
25.consider the following statements about temperate cyclones?

1) They have a low-pressure gradient than that of tropical cyclones

2) They travel from east to west

3) They are generally produced in summers

Which among the following above statements is/are correct?

A. Only 1
B. Only 2
C. Only 1 and 3
D. All are correct

Answer ||| A

Solution |||
Temperate Cyclones are produced in winters, and they travel from West to East.

Temperate Cyclones, also known as mid-latitudinal cyclones or Extratropical Cyclones, are


created due to interaction of air masses of various temperature. The cold air mass come from
the polar region & hot air mass from the temperate region. Fronts are the interaction zone of 2
cyclones.

Characteristics of Temperate Cyclones:

* A feature of the tropical cyclones is the eye formation, a central region of clear skies, warm
temperatures & low atmospheric pressure. Frequently, atmospheric pressure at the surface of
Earth is around 1,000 millibars.

* They travel from west to east.

* They are formed in the winter season.

* The temperate cyclones are asymmetrical and shaped like an inverted 'V'.

* They have a height of 8 - 11 km.

* They stretch over 500 - 600 km.

* They may spread over 2500 km comprises over North America (Polar Vortex).

26.Match the following local winds and their location:


Choose the correct answer from the given code:

A. a-1 b-2 c-3 d-4


B. a-1 b-2 c-4 d-3
C. a-4 b-5 c-1 d-3
D. a-3 b-5 c-1 d-4

Answer ||| B

Solution |||
27.Consider the following statement/s with reference to the humidity in the air:

1. The amount of water vapour present in the air depends on the temperature.

2. Relative Humidity is inversely proportional to temperature.

Choose the correct answer from the given codes:

A. 1 only
B. 2 only
C. Both 1 and 2
D. Neither 1 nor 2

Answer ||| C
Solution |||

Absolute Humidity is the total amount of the water Vapor present in the atmosphere and is
the weight of water vapour per unit volume of air.

Absolute Humidity or the ability of the air to hold water vapour is directly proportional to
temperature; thus, it differs from place to place on earth.

Relative Humidity is the percentage of moisture present in the atmosphere as compared to its
full capacity to hold moisture at a given temperature. Relative humidity has inverse
proportionality with temperature.

28.Consider the following pairs:

Dam Projects -Rivers

1) Srisailam -Krishna

2) Krishnarajasagar -Godavari

3) Mettur -Kaveri

Which of the pairs is/are correctly matched?

A. 1 and 2 only
B. 2 and 3 only
C. 1 and 3 only
D. 3 only

Answer ||| C

Solution |||

Srisailam Dam: It is constructed across the Krishna River in Kurnool district, Andhra
Pradesh and is the 2nd largest capacity working hydroelectric station of India.

Krishna Raja Sagar Dam: Named after Krishnaraja Wodeyar IV of Mysore, this dam is
built over the River Cauvery/Kaveri; near to the conjunction of three rivers namely, Cauvery,
Hemavathi & Lakshmana Thirtha. A major source of drinking water for Mysore and
Bangalore city, K.R.S dam is also one of the main sources of irrigation water for Mandya and
Mysore. Along with this, it major purpose is to ensure power supply to the Shivanasamudra
hydroelectric power station. With a length of 3 km, this dam also boasts of being the first
irrigation dam of India.

Mettur Dam: It is the oldest dam of India located across the Cauvery river in Salem District
of Tamilnadu. It is also the largest dam of Tamilnadu. It gained public attention due to the
Kaveri River water dispute between the States of Tamil Nadu and Karnataka.
29.Consider the following statements with reference to the incoming solar radiation:

1) The insolation received by the Earth is more at the perihelion than at the aphelion.

2) The equator receives comparatively less insolation than the tropics.

3) At the same latitude, the insolation received is more over the oceans than over the
continents.

Which of the statements given above is/are correct?

A. 2 and 3 only
B. 1 and 2 only
C. 1 only
D. 2 only

Answer ||| B

Solution |||

• Statement 1 is correct:

The solar output received at the top of the atmosphere varies slightly in a year due to the
variations in the distance between the Earth and the sun. During its revolution around the sun,
the Earth is farthest from the sun (152 million km) on 4th July. This position of the Earth is
called aphelion. On 3rd January, the Earth is the nearest to the sun (147 million km). This
position is called the perihelion. Therefore, the annual insolation received by the Earth on 3rd
January is slightly more than the amount received on 4th July. However, the effect of this
variation in the solar output is masked by other factors like the distribution of land and sea
and the atmospheric circulation.

• Statement 2 is correct:

The insolation received at the surface varies from about 320 Watt/m2 in the tropics to about
70 Watt/m2 in the poles. Maximum insolation is received over the subtropical deserts, where
the cloudiness is the least. The Equator receives comparatively less insolation than the
tropics.
• Statement 3 is not correct:

Generally, at the same latitude, the insolation is moreover the continent than over the oceans
In winter, the middle and higher latitudes receive less radiation than in summer.

30.Marawah Island recently in news is a territory of-

A. United Arab Emirates


B. Qatar
C. Iran
D. Indonesia

Answer ||| A

Solution |||

Recently the world’s oldest known Natural Pearl (dubbed the ‘Abu Dhabi Pearl’) has been
discovered by Abu Dhabi Archaeologists at a Neolithic site on Marawah Island, just off the
coast of Abu Dhabi, the capital city of United Arab Emirates. The Pearl has been found to be
more than 8,000 years old dated back to the Neolithic period

Other significant findings from these sites include imported Ceramic Vase from Ubaid
civilisation in Mesopotamia, beautifully worked flint Arrowheads and shell and stone beads.

31.Consider the following statements pertaining to Coffee plantation in India :

1) Need warm and moist climate with a spell of dry weather during the ripening period

2) Rolling fields having good drainage


3) Strong sunshine over hilly slopes exceeding temperature 35 oC

4) Karnataka is the leading producer in India

Which of the statements given above are correct?

A. 1 and 4 only
B. 1, 2 and 3
C. 3 and 4
D. 1, 2 and 4

Answer ||| D

Solution |||

Coffee plantation in India needs a hot and humid climate having a temperature between 15 0c
to 280 C. It requires rainfall between 150 to 250 cm.

The ideal soil for coffee plantation is rolling surface and which is easily workable.

Karnataka is a leading producer for coffee plantation in India. The state almost contributes
70% of Coffee plantation in India.

32.Which of the following industries should be located near the source of Raw Material?

1) Cement Industry

2) Woolen Industry

3) Jute Industry

4) Sugar Industry

5) Paper Industry

Choose the correct answer from the codes given below.

A. 1, 3 and 4 only
B. 1, 4 and 5 only
C. 3, 4 and 5 only
D. 2, 4 and 5 only

Answer ||| B

Solution |||
The raw material for the Woolen industry – wool, is non-perishable, lightweight, and the
finished good does not lose weight or gets affected during transportation. Therefore for the
Woollen Industry, the proximity to raw material site is not essential. Major factors considered
for the location of a Woollen Sector are - Proximity to Market, Cheap Electricity to run
power looms and machines, Cheap labour supply.

Raw material for the Cement Industry are- Limestone (most important), Clay (silica,
alumina) and other supplementary materials such as Sand, Fly Ash/Pulverised Fuel Ash
(PFA). Cement Industry could have cost advantages if, it should have access to a quality
limestone reserve, a cheaper energy source, and proximity to bigger markets.

Raw material for Jute Industry is Jute, other important considerable factors for locating a
jute factory are - a large quantity of water for Washing, Bleaching & Retting, Cheap labour
and cheap energy source. All of the factors are equally important, and anyone can be the
deciding factor.

Raw material for Sugar Industry is Sugarcane. It is a perishable commodity as once the
sugarcane is cut; its sucrose content starts to decline. Also, 100kg of Sugarcane yields only
approx. 10 kgs of Sugar. Therefore considering these factors, the Sugar Industry is located
near the raw material source.

Raw material which is widely used by Paper Industry is Pulp. More than 90% of the total
manufactured paper is made from wood pulp today. Wood pulp is obtained by chemically or
mechanically reducing wood fibre into a lignocellulosic fibrous material. Thus the paper
industries are located near to the raw material to minimises the difficulty and cost of
transporting the bulk logs of wood. A river-side site is ideal because logs can be floated
directly to the mill.

33.Uranium is found in which of the following locations in India?

1) Chhattisgarh

2) Andhra Pradesh

3) Rajasthan

Select the correct answer using the code given below.

A. 1 only
B. 1 and 2 only
C. 2 and 3 only
D. 1, 2 and 3

Answer ||| D

Solution |||
Uranite or pitchblende mineral of Uranium is found in Gaya (Bihar), Ajmer (Rajasthan),
Durg (Chhattisgarh) and Nellore (Andhra Pradesh).

India’s major Uranium deposits are shown in the following diagram -

34.Consider the following pairs:

Which of the pairs given above is/are correctly matched?

A. 1, 2 and 3
B. 2 only
C. 3 only
D. 1 and 3 only
Answer ||| C

Solution |||

Zoji La is located on the Great Himalayas, Banihal on the Pir Panjal, Khardung La on the
Ladakh range.

Following map shows the ‘Passes of Northern India’ -

35.Match List-I with List-II and select the correct answer using the code given below in the
Lists:

List-I (Place) - List-II (Mineral)

A) Lakwa - 1) Copper

B) Malanjkhand - 2) Petroleum

C) Kalakot - 3) Zinc

D) Zawar - 4) Coal

A. 4 - 3 - 2 - 1
B. 4 - 2 - 3 - 1
C. 1 - 2 - 3 - 4
D. 1 - 3 - 2 - 4
Answer ||| C

Solution |||

• Petroleum is found in India in Guwahati, Barauni, Koyali, Haldia, Mathura, Digboi, Lakwa,
Panipat, Chennai, Narimanam, Bongaigaon, Mumbai (HPCL), Vishakhapatnam, Mumbai
(BPCL), Kochi, Numaligarh, Tatipaka (ONGC) and Bina (M.P.)

• Copper is found in India in Singhbhum district (Jharkhand), Balghat district (Madhya


Pradesh), Malanjkhand (Madhya Pradesh), Jhunjhunu and Alwar districts (Rajasthan). In
addition, there are little deposits in Gujarat, Andhra Pradesh, Karnataka, Uttar Pradesh,
Sikkim, Meghalaya, Maharashtra and West Bengal.

• Zinc is found in India in, most of the resources are found in Rajasthan. A lesser amount of
resources are available in Andhra Pradesh, Madhya Pradesh, Bihar and Maharashtra states.
Some deposits containing zinc are even found in Udhampur district of Jammu & Kashmir,
Zawar (Distt. In Udaipur, Rajasthan) and South Arcot Vallalar district of Tamil Nadu.

• Coal is found in India in Ranigunj (Oldest coalfield in India) in West Bengal, Jharia
(Largest), Bokaro, Dhanbad, Giridih, Karanpura, Ramgarh, Daltonganj in Jharkhand,
Singrauli, Suhagpur, Johila, Umaria, Satpura coalfield in Madhya Pradesh, Kalakot (a village
in Jammu & Kashmir), Talcher, Himgiri, Rampur in Odisha, Korba, Bisrampur, Sonhat,
Jhilmil, Hasdo-Arand in Chattisgarh, Makum, Najira, Janji, Jaipur in Assam etc.

36.Which one of the following States does NOT have the Headquarters of any Railway
Zone?

A. Jharkhand
B. Chhattisgarh
C. Odisha
D. Bihar

Answer ||| A

Solution |||
37.Arrange the following states in order of their decreasing population size as per the census
2011 :

1) West Bengal

2) Bihar

3) Maharashtra

4) Andhra Pradesh

Select the correct answer using the code given below :

A. 2 – 3 – 1 – 4
B. 3 – 2 – 1 – 4
C. 1 – 4 – 3 – 2
D. 3 – 1 – 4 – 2

Answer ||| B

Solution |||

As per the Census of India 2011, the population distribution is as below:


Uttar Pradesh- 9.28% ; Bihar- 8.6% ; West Bengal- 7.54% ; Andhra Pradesh- 6.99%

38.Match List I with List II and select the correct answer using the code given below the Lists
:

List I(River)

A) Danube

B) Rhine

C) Rhone

D) Loire

List II(Mouth of the river)

1) North sea

2) Black sea

3) Bay of Biscay

4) Mediterranean sea

A. A-2 B-1 C-4 D-3


B. A-3 B-4 C-1 D-2
C. A-2 B-4 C-1 D-3
D. A-3 B-1 C-4 D-2

Answer ||| A

Solution |||

A) Danube - Black sea

B) Rhine - North sea

C) Rhone - Mediterranean sea

D) Loire - Bay of Biscay

The Danube discharges into the Black Sea, located in Central and Eastern Europe.

The Rhine is one of the major European rivers, which has its sources in Switzerland and
flows in a mostly northerly direction through Germany and the Netherlands, emptying into
the North Sea.
The Rhône is one of the major rivers of Europe and has twice the average discharge near its
mouth on the Mediterranean Sea, the river divides into two branches, known as the Great
Rhône.

The Loire is the longest river in France and the 171st longest in the world. With a length of
1,012 kilometres (629 mi), it drains an area of 117,054 km2 The main rivers that empty into
the Bay of Biscay are Loire, Charente, Garonne, Dordogne, Adour, Nivelle, Bidasoa,
Oiartzun, Urumea, Oria, Urola, Deba etc.

39.Which one of the following is the correct descending order of countries in terms of per
capita availability of arable land?

A. India – China – Brazil


B. China-Brazil – India
C. Brazil – China – India
D. Brazil – India – China

Answer ||| D

Solution |||

Arable land includes land defined by the FAO as land under temporary crops (double-
cropped areas are counted once), temporary meadows for mowing for pasture, land under
market or kitchen gardens, and land temporarily fallow.

Hectares per area of arable land (2011 data) is as below:

Brazil- 2.14 ;

India- 0.129 ;

China- 0.083

40.Which organization prepares the topographical maps of India?

A. Geological Survey of India


B. Archaeological Survey of India
C. Survey of India
D. National Atlas and Thematic Mapping Organisation

Answer ||| C

Solution |||
Survey of India is India’s central engineering agency which prepares topographical maps of
India.

41.Which one of the following is not an international boundary line?

A. Dresden Line
B. Durand Line
C. Maginot Line
D. 38th Parallel

Answer ||| A

Solution |||

Dresden Line- It is a network of tramway forming public transportation system in Dresden, a


city in Germany.

42.Which one of the following statements is not correct?

A. Temperatures decrease from the equator to poles


B. Temperatures in equatorial regions change substantially from January to July
C. Large landmasses located in the subarctic and arctic zones develop centres of extremely
low temperatures in winter
D. Highlands are always colder than surrounding low lands

Answer ||| B

Solution |||

Temperature decreases while going from the equator towards the poles because the sun is less
far from the equator and receives direct sunlight. As we go towards the poles, the sunlight
will decrease.

Mid-day sun is almost overhead throughout the year, and there is very little difference
between the lengths of day and night. The average temperature varies almost 2.2 0C. The
climate is temperate and warm with rain throughout the year.

Sub-Arctic Climate is found over the large landmasses. These land masses are away from
the effect of the ocean and located almost latitude from 500 to 700. This climate or
landmasses are the sources of cold air that impacts temperate latitude to the south in winter.

Highland is always cooler than low lands. It is because temperature decreases with increase
in elevation.
43.Match List I with List II and select the correct answer using the code given below the
Lists:

Code:

A. A-2 B-3 C-4 D-1


B. A-1 B-4 C-3 D-2
C. A-2 B-4 C-3 D-1
D. A-1 B-3 C-4 D-2

Answer ||| A

Solution |||

Dudhwa: The Dudhwa National Park is a national park in the Terai belt of marshy grasslands
of northern Uttar Pradesh, India.

Bhitarkanika: Description Bhitarkanika Mangroves is a mangrove wetland and tiger reserve


in Odisha, India, covering an area of 650 km in the Brahmani and Baitarani river deltas.

Chilka: Chilka Lake is Asia's largest salted water lake located in Orissa, India and also
known as Chilka Lake Bird Sanctuary.

Nokrek: Nokrek National Park, the core area of Nokrek Biosphere Reserve, is a national park
located approximately 2 km from Tura Peak in West Garo Hills district of Meghalaya, India.
UNESCO added this National park to its list of Biosphere Reserves in May 2009.

44.Temperate Cyclones occur mostly in winter, late autumn and spring, generally associated
with rainstorms and cloudy weather. Which of the following statements is/are true about
Temperate Cyclones -

A. Formation can form both on land as well as seas.


B. The shape of the cyclone is elliptical
C. Origin of Temperate Cyclone is due to Coriolis Force and movement of Air Masses.
D. Temperate Cyclones can be seen in India in winter.

Answer ||| B
Solution |||

In the Northern Hemisphere, warm air moves from the South and cold air move from the
front to the North. When the pressure falls to the front, the warm air moves Northward and
the cold air moves southward at speed in the cyclonic momentum circulation (Northern
Hemisphere) due to the Coriolis force.

The Cold Front moves hot air from behind and pushes hot air up and cumulus clouds develop
along the cold front. Cold fronts move faster than warm fronts and eventually overtake warm
fronts. The hot air is completely lifted up and vented to the front (front side) and the cyclone
spreads. The shape of the cyclone is inverted ‘V’. Temperate Cyclones can be seen in the
United States and Canada.

45.

|||Common||| Directions: The following seven (7) items consist of two statements, Statement
I and Statement II, You are to examine these two statements carefully and select the answers
to these items using the code given below: |||End|||

Statement I: Grand Banks are one of the major fishing grounds of the world due to the
presence of a vast continental shelf.

Statement II: Planktons grow in the shallow waters.

A. Both the statements are individually true and Statement II is the correct explanation of
Statement I
B. Both the statements are individually true but Statement II is not the correct explanation of
Statement I
C. Statement is true but Statement II is false
D. Statement I is false but Statement II is true

Answer ||| A

Solution |||

The Grand Bank is an area of submerged highlands. It is situated in the east of Laurentian
Channel and South-East of Newfoundland on the continental shelf of North America. In this,
cold Labrador Current and warm Gulf Stream mixes up. Because of this mixing and shape of
the continental shelf, nutrients lifts up from the ocean bottom to the surface. Because of all
these circumstances, one of the biggest fishing ground of the world will get created here.

Planktons are organisms that live in water and usually found in shallow water. They are often
carried by tides, winds and currents.
46.Statement I: It is not necessary that every bar magnet has one North Pole and one South
Pole.

Statement II: Magnetic poles occur in pair.

A. Both the statements are individually true and Statement II is the correct explanation of
Statement I
B. Both the statements are individually true but Statement II is not the correct explanation of
Statement I
C. Statement is true but Statement II is false
D. Statement I is false but Statement II is true

Answer ||| D

Solution |||

Every bar magnet has two poles which are the North Pole and the South Pole. Every magnet
must have two poles.

Because every bar magnet exists in a pair of two-pole (North and South) with the help of the
compass.

47.Mulching, an agronomic measure of soil conservation, is very effective because it -

1) protects soil from gully erosion

2) protects soil from sheet wash and wind erosion

3) helps soil to retain moisture and nutrients

Select the correct answer using the code given below

A. 1 only
B. 1 and 2 only
C. 2 and 3 only
D. 1, 2 and 3

Answer ||| C

Solution |||

Mulching is a conservational practice implemented on the soil to preserve the health of the
soil. Mulch is a protective layer of organic material (like- bark of tree, wood chips, grape
pulp, shell nuts, green waste, leftover crops, compost, manure, straw, dry grass, leaves etc.)
that is spread on top of the soil. Mulches can also be inorganic material like stones, brick
chips, and plastic. Both organic and inorganic mulches have numerous.
Benefits of Mulching:

* Protects the soil from erosion

* Reduces compaction from the impact of heavy rains

* Conserves moisture thus reducing the need for frequent irrigation.

* Maintains a more even soil temperature

* Prevents weed growth

* Keeps crops, fruits and vegetables clean

* Organic mulches also improve the condition of the soil. As these mulches slowly
decompose, they provide organic matter which helps keep the soil loose. This improves root
growth, increases the infiltration of water, and also improves the water-holding capacity of
the soil. Organic matter is a source of plant nutrients and provides an ideal environment for
earthworms and other beneficial soil organisms.

48.Arrange the following Indian cities according to their locations from West to East:

1) Bilaspur

2) Jodhpur

3) Bhopal

4) Ranchi

Select the correct answer using the code given below:

A. 3-2-1-4
B. 2-3-1-4
C. 4-1-2-3
D. 2-1-3-4

Answer ||| B

Solution |||
49.Which of the following is not a Causative Factor for Ocean Current?

A. Earth’s Rotation
B. Density Difference
C. Planetary Winds
D. All are causative factors

Answer ||| D

Solution |||

Causative Factors are responsible for the generation of Ocean Currents. They are-

* Earth’s Rotation- Ocean Currents follow the clockwise direction in the Northern
Hemisphere and Anticlockwise direction in the Southern Hemisphere.

* Planetary Winds- they drive the Ocean Currents in a steady flow in front of them.

* Density Difference- higher the temperature, lesser will be the density of the water. Hence,
water with low salinity flows to the water of high salinity while an undercurrent of high
salinity flows towards the less dense water.

50.Which of the following currents are Cold Currents?

1) Kuroshio

2) Aghulas
3) California

4) Alaska

Choose the right code-

A. 3 only
B. 3 and 4
C. 1, 3 and 4
D. 1 and 4

Answer ||| A

Solution |||

California Current along the west coast of USA is a Cold Current. Rest all are Warm
Currents.

Kuroshio Current - Along the East coast of Japan. (Pacific Ocean)

Agulhas Current - Along the East Coast of Africa

Alaska Current - Alongside the West Coast of North America.

51.Match List-I with List-II and select the correct answer using the code given below the
Lists :

List –I (Hydroelectric Power Station)


A. Srisailam

B. Sabarigiri

C. Hirakud

D. Sileru

List – II (Location in the map)

A. A-3 B-1 C-4 D-2


B. A-3 B-4 C-1 D-2
C. A-2 B-4 C-1 D-3
D. A-2 B-1 C-4 D-3

Answer ||| B

Solution |||

List of some important hydroelectric Project and their location:

1) Tehri Dam: Uttarakhand

2) Koyna Hydroelectric Project: Maharashtra

3) Srisailam: Andhra Pradesh

4) Nathpa Jhakri: Himachal Pradesh

5) Sardar Sarovar Dam: Navagam, Gujarat

6) Bhakra Nangal Dam(Gobind Sagar):Himachal

7) Chamera I: Himachal Pradesh


8) Sharavathi Project: Karnataka

9) Indira Sagar Dam: Madhya Pradesh

10) Karcham Wangtoo Hydroelectric Plant: Himachal Pradesh

11) Dehar (Pandoh) Power Project: Himachal Pradesh

12) Nagarjuna Sagar Dam Guntur: Andhra Pradesh

13) Purulia Pass: West Bengal

14) Idukki: Kerala

15) Salal I & II: Jammu & Kashmir

16) Upper Indravati: Orissa

17) Ranjit Sagar Dam: Punjab

18) Omkareshwar: Madhya Pradesh

19) Belimela Dam: Orissa

20) Teesta Dam: Sikkim

52.Consider the following statements :

Statement I : Clay layers are poor aquifers

Statement II : The inter-particle space of clay minerals is the least

Select the correct answer using the code given below :

A. Both the Statements are individually true and Statement II is the correct explanation of
Statement I.
B. Both the Statements are individually true but Statement II is not the correct explanation of
Statement I
C. Statement I is true but Statement II is false
D. Statement I is false but Statement II is true

Answer ||| A

Solution |||
An aquifer is a body of rock or unconsolidated sediments that has sufficient permeability to
allow water to flow through it. Clay soils do not make good aquifers as the it is less
permeable due to small particle size. Which are tightly packed together.

53.Which one of the following is not a tributary of River Krishna?

A. Bhima
B. Purna
C. Tungabhadra
D. Malaprabha

Answer ||| B

Solution |||

Purna is a tributary of river Godavari while rest all are tributaries of river Krishna.

54.Recently Hurricane Dorian hit Lucayan Archipelago, which of the following Country was
most affected by it?

A. Costa Rica
B. Papua New Guinea
C. Bahamas
D. Mauritius
Answer ||| C

Solution |||

Hurricane Dorian was a Category 5 Hurricane that caused ;large scale destruction in the
Bahamas. India had announced immediate humanitarian assistance of $1 million to help
people in The Bahamas affected by Hurricane Dorian.

55.Recently Bougainville Island became a country. Island is a part of -

A. Soloman Island
B. Vietnam
C. Philippines
D. Indonesia

Answer ||| A

Solution |||

On December 11, 2019, Bougainville, the largest island of the Solomon Islands Archipelago
voted to become independent of Papua New Guinea. Around 98% of people voted in favour
of independence.

56.Rice is a

A. Kharif crop
B. Rabi crop
C. Zaid crop
D. Both A and B

Answer ||| A

Solution |||

Important Kharif crops are: rice, maize, sorghum, pearl millet/bajra, finger millet/ragi
(cereals), arhar (pulses), soyabean, groundnut (oilseeds) and cotton.

Kharif crops are usually sown with the beginning of the first rains during the advent of the
south-west monsoon season, and they are harvested at the end of monsoon season (October-
November).

Important Rabi crop : wheat, barley, oats (cereals), chickpea/gram (pulses), linseed and
mustard (oilseeds).
Important Zaid crop : watermelon, pumpkin, cucumber, bitter groud and Sunflower.

57.Consider the contour plot given below:

Contour in Meters

The above contours of an area indicate several relief features. Which one among the
following relief features is not depicted here?

A. Steep slope
B. River valley
C. Conical hill
D. Gentle slope

Answer ||| C

Solution |||

Contour line is a line on a map joining points of equal height above or below sea level.

A Conical Hill is a landform with a distinctly conical shape. It is usually isolated or rises
above other surrounding foothills. A conical hill is represented by concentric contours spaced
at almost regular intervals. In the given contour plot, concentric contours are absent.
Concentric contours of conical hills are given below:
58.Match List I with List II and select the correct answer using the code given below the
Lists:

List-I (Type of Grass)

a) Llanos

b) Prairies

c) Pampas

d) Downs

List-II (Country)

1) Australia

2) Venezuela

3) USA
4) Argentina

A. a-1 b-4 c-3 d-2


B. a-1 b-3 c-4 d-2
C. a-2 b-4 c-3 d-1
D. a-2 b-3 c-4 d-1

Answer ||| D

Solution |||

Grasslands are found where there is not enough regular rainfall to support the growth of a
forest, but not so little that a desert forms. In fact, grasslands often lie between forests and
deserts.

• Llanos is a tropical grassland situated in Colombia and Venezuela.

• Prairies are temperate grasslands located in USA.

• Pampas are temperate grasslands located in Argentina.

• Downs are grasslands located in Australia.

59.Why is the lower course of a river in many cases filled with sediments?

1) Erosion dominates in the lower course of the river.

2) The velocity of a river is low in its river course.

Which of the above is /are correct.

A. Only 1
B. Both are correct
C. Only 2
D. None of these

Answer ||| C

Solution |||

Erosion dominates in the upper course where river velocity is sufficiently high. This increases
the sediment load of the river, which reaches the lower course is dumped by the river.

The sediment carried is dumped only when the river velocity is low enough for suspended
particles to settle down, and erosion activity considerably reduces, this happens in the lower
course of the river.
60.Match List-I with List-II and select the correct answer using the code given below the
Lists:

A. a-4 b-1 c-2 d-3


B. a-4 b-2 c-1 d-3
C. a-3 b-2 c-1 d-4
D. a-3 b-1 c-2 d-4

Answer ||| A

Solution |||

Ebony is found in tropical evergreen forests; Shisham in moist deciduous; Walnut in


himalayan moist forests and Birch in temperate, boreal and alpine climates.

61.Which of these is the correct order of these regions with regards to the incidences of
landslides occurring in them.

A. Vidyanchal > North Eastern hills > Himalaya> Western ghats and the nilgiris.
B. Himalayas > North Eastern hills > Western ghats and the nilgiris > vidyanchal
C. Himalayas > vidyanchal > Western Ghats and the nilgiris > North Eastern hills.
D. Western Ghats and the Nilgiris > Vidyanchal > North Eastern hills > Himalayas.

Answer ||| B

Solution |||

Depending on the rock structure, different regions have varying incidences of landslides.

Soft and unconsolidated formations like Himalayas have high to very high incidence of
landslides.

North Eastern hills to have high incidences.

Western Ghats and the Nilgiris have a moderate to low and Vidyanchal has low incidence
due to the hard and consolidated Rock structure.
62.Sir Creek is a disputed area between which of the following countries?

A. India-Afghanistan
B. India-Pakistan
C. India-Myanmar
D. India-Bangladesh

Answer ||| B

Solution |||

Sir Creek ia a disputed area in Rann of Kutch . Sir Creek is a 96 kilometres long estuary in
the marshes of the Rann of Katch where Arabian Sea joins the land mass. Sir Creek is
considered to be one of the largest Fishing grounds in Asia. It also has potential economic
benefits as the marshlands are estimated to be rich in hydrocarbons and shale gas, is another
importance of Sir Creek

63.Which of the following statement is wrong?

A. Nitrogen is very rare in Earth’s atmosphere.


B. The Pacific plate is the fastest-moving of the Earth’s plates.
C. Earth has geomagnetic poles.
D. Earth’s Stratosphere is very cold.

Answer ||| A

Solution |||

Nitrogen composes about 78 percent of the lower atmosphere. Oxygen, Argon, and other
gases compose the rest. The Pacific plate, which underlies most of the Pacific Ocean, is
moving at a rate of about 7.2 inches (18.3 centimeters) a year. Most of the other plates are
about half as fast.

Our planet has two magnetic poles, one in the Arctic and one in the Antarctic. They reverses
their polarity, and no one is quite sure why this happens. Reaching to 50 miles (80
kilometres) away from Earth, the Stratosphere is very cold: it is about –148 ºF, or –100 ºC.

64.The sediments deposits at the oceans floors derived from various sources. Which of the
following are Terrigenous Deposits –

A. Gravel, Sand, Silt


B. Ash, Pumice, Cinder
C. Dust, Tektites and Precipitation
D. Neritic deposit, Globigerina ooze

Answer ||| A

Solution |||

Terrigenous deposits are derived from the wear and tear of land and volcanoes and organic
products. The bulk of the deposits on the continental shelf and slopes are derived from rock
material that is loosened by disintegration and dissolution by agents of weathering and
carried to the sea by agents of erosion, such as flowing water, wind, etc.

Terrigenous deposit can be classified into three classes that are soil, sand and gravel. Soil
refers to those fine particles, consisting mainly of minute particles of rock-forming mineral in
quartz. Murray has classified Mud deposits into blue, green and red types based on the colour
of the constituent. Sand refers to coarse particles, while Gravel contains even larger particles.

65.Which of the following line indicates points of equal rainfall on a map:

A. Isohel
B. Isobar
C. Isohyet
D. Isotherm

Answer ||| C

Solution |||

• Isohel is a line of equal or constant solar radiation.

• Isobar is a line connecting places of same atmospheric pressure.

• Isohyet is a line joining points of equal rainfall on a map in a given period.

• Isotherm is a line that connects points on a map that have the same temperature.

66.With reference to Black Soils, which of the following statements is not correct:

A. These soils are also called Regur soil


B. They are generally found in Northern India
C. They are good for cultivation of cotton
D. They are rich in iron
Answer ||| B

Solution |||

• Black soils are also known as the ‘Regur Soil’. The black soils are generally clayey, deep
and impermeable. They can retain moisture which is required for cultivation of cotton. Hence
they are suitable for cultivation of cotton.

• Chemically, the black soils are rich in lime, iron, magnesia and alumina. They also contain
potash.

• Black soil covers most of the Deccan Plateau which includes parts of Maharashtra, Madhya
Pradesh, Gujarat, Andhra Pradesh and some parts of Tamil Nadu.

67.Consider the following statements, which of the following is not true?

A. Asthenosphere is the main source of magma.


B. The rate of increase in temperature below the surface is decreasing towards the centre
C. Lithosphere is divided into tectonic plates
D. The temperature is not always increasing from the surface of the Earth towards the centre.

Answer ||| D

Solution |||

Whereas in the upper 100kms, the temperature increases at a rate of 120C per km and in the
next 300kms, it is 200C per km. But going further deep, this rate decreases to just 100C per
km. Thus, it is assumed that the rate of increase in temperature below the surface is
decreasing towards the centre (do not confuse the rise of temperature with the increase in
temperature. The temperature is always increasing from the surface of the earth towards the
centre). .

The top portion of the crust and mantle is called the lithosphere. Its thickness ranges from
10–200 km. The lithosphere is divided into tectonic plates.

The upper part of the mantle is called the asthenosphere, which extends up to 400 km. The
word astheno means weak. It is the main source of magma that finds its way to the surface
during a volcanic eruption. It lies beneath the lithosphere.

68.El-Nino:

1) El-Nino is a climate pattern that’s occur in Eastern Tropical Pacific Ocean

2) positively affects the Indian monsoon


Choose the correct option:

A. 1 only
B. 2 only
C. 1 and 2
D. Neither of the two

Answer ||| A

Solution |||

• El-Nino is a climate pattern that’s occur in Eastern Tropical Pacific Ocean that affects
weather all over the world. It is a temporary replacement of the cold Peru Current which
normally flows along this coast.

• El-Nino increases the surface water temperatures of the sea by 10° C. El Nino affects the
flow of moisture-bearing winds from the cooler oceans towards India and negatively impacts
the summer monsoon, which accounts for over 70% of annual rainfall. It is believed that the
severest drought of 1987 over India was caused by EI-Nino.

69.Consider the following

I. New Guinea

II. Madagascar

III. Borneo

IV. Greenland

Which of the following is the correct chronological order according to decreasing area?

A. I-II-III-IV
B. II-IV-III-I
C. II-III-I-IV
D. II-I-IV-III

Answer ||| B

Solution |||

1. Greenland

* Location: North Atlantic (Danish)

* Area (s km): 2,175,597


2. New Guinea

* Location: Southwest Pacific (West Papua [Irian Jaya], Indonesia, western part; Papua New
Guinea, eastern part)

* Area (s km): 800,311

3. Borneo

* Location: West mid-Pacific (Indonesian, south part; Brunei and Malaysian, north part)

* Area (s km): 744,108

4. Madagascar

* Location: Indian Ocean (Malagasy Republic)

* Area (s km): 587,931

70.Which one among the following is the most plausible explanation for the location of the
Thar Desert in Western India?

A. Prevalence of sand dunes

B. The evaporation of moisture by heat

C. The absence of mountains to the North of Rajasthan to cause orographic rainfall in it

D. The moisture carried by the South-West Monsoon is driven away by the dry upper air
current

Answer ||| C

Solution |||

Thar desert is located in the north-western part of India in the state of Rajasthan. It is
world’s ninth largest subtropical desert. Strong winds lifted the sand and silt from alluvial
sediments and deposited them in this region. Absence of any mountain range is one of the
reasons for the formation of Thar Desert. As the mountain blocks the moisture containing
winds and cause rainfall (called orographic rainfall).

71.Consider the following statements about ‘El Nino’:

1) It is the periodic development of a cold ocean current along the coast of Peru as a
replacement of warm Peruvian current.
2) Presence of this current leads to a decrease in sea surface temperature and hence
weakening of the trade winds in the region.

Which of the statements given above is/are correct?

A. 1 only
B. 2 only
C. Both 1 and 2
D. Neither 1 nor 2

Answer ||| D

Solution |||

El Niño is complex weather phenomenon resulting from variations in ocean temperatures in


the Equatorial Pacific region.

Along the coast of Peru, a warm ocean current is replacing the cold Peruvian current is the
phenomenon of El-Nino.

Presence of this current leads to an increase in sea surface temperature and hence weakening
of the trade winds in the region. El Nino is a Spanish word meaning ‘child’ referring to the
baby Christ as it starts flowing during Christmas.

72.Which of the following factors influence ‘South-West Monsoon’ in India?

1) Intense heating of Tibetan plateau

2) Westerly Jet Stream

3) Tropical Easterly Jet Stream

Select the correct answer using the code given below:

A. 1 only
B. 2 and 3 only
C. 1 and 3 only
D. 1, 2 and 3

Answer ||| D

Solution |||

Various factors influence South-West Monsoon in India. The South west monsoon strikes at
the Kerala coast by 1 st June and then move towards Mumbai and Kolkata between 10 th and
13th June.
The Tibetan plateau is intensely heated during summer and thereby establishes an
atmospheric circulation that is conducive for the monsoon. As the vast Tibetan plateau, high
up in the mountains, gets warmed up during the summer months, it heats the air above. Such
warm air rises and hence creates an area of low pressure over Tibetan Plateau. This belt of
low pressure sucks in moisture laden winds from the oceans, thus initiating the south-west
monsoon.

Sub-tropical Westerly Jet Stream is responsible for the onset of the monsoon. Until the end of
May, this jet stream supplies the wind to the Gangetic plain which is heated up intensely as
the sun is directly above the Tropic of Cancer (during the summer in Northern hemisphere).
These winds supplied by this jet stream prevent the formation of a low pressure area in the
Gangetic plain. When sub-tropical westerly jet stream moves towards north by the end of
May, it no longer provides the winds to the Gangetic plain. This results in the formation of a
Low Pressure area in the Gangetic plain, which starts attracting the South East Trade winds
of the southern hemisphere. When such winds enter the northern hemisphere, they bend by 90
degrees due to the change in direction of the Coriolis force in the northern hemisphere. This
marks the onset of the South West Monsoon.

Tropical depression is made because of easterly jet stream. This is having important role in
monsoon rainfall over the Indian subcontinents.

73.Consider the following statements and choose the correct option.

1) Chenab originate from confluence of two rivers, the tapi and Beas.

2) Jhelum originates from South Eastern part of Kashmir.

3) Ravi originates from Rohtang pass in Himalayas.

4) Satluj originates from Rekas lake.

Codes-

A. 1 and 2
B. 1, 2 and 3
C. 2, 3 and 4
D. All of these

Answer ||| C

Solution |||

* Chenab River is a major river which flows in India and Pakistan and is one of the five
major rivers of Punjab region. The river is formed at the confluence of two rivers, Chandra
and Bhaga.
* Jhelum River is a river in Northern India and East Pakistan, it is the westernmost of the five
rivers of the Punjab region and passes through the Kashmir Valley.

* Ravi river is a transboundary river that crosses northwestern India and eastern Pakistan. It is
one of the six rivers of the Indus system in the Punjab region. Under the Indus Water Treaty,
Ravi's water is allocated to India.

* The Sutlej River is the longest of the five rivers flowing through the historic crossroads area
of Punjab in northern India and Pakistan. This river is also known as Satadri.

74.Which one of the following is not an agent of metamorphism?

A. Heat
B. Compression
C. Decomposition
D. Solution

Answer ||| C

Solution |||

Change in structure, texture and composition of rock due to the impact of different agents is
called as a metamorphism. There are three agents of metamorphism heat,
pressure(compression) and fluid action (solution).

Most common agent is hear because it provide energy to drive the changes or
recrystallization of minerals. Pressure is the component, as you go in depth it will increase
because of load above. These stresses and tension will shorten the rock body. Most common
fluid as metamorphic agent is water. Water us having ions to form solution and also water
can contained in the pore spaces of every rock.

75.Which one of the following statements about temperature is correct?

A. Temperature decreases with height in the stratosphere


B. Temperature is constant at different heights in the stratosphere
C. Temperature increases with height in the troposphere at an average rate of 6.5 C per
kilometer
D. Temperature decreases with height in the troposphere at an average rate of 6.4 C per
kilometer.

Answer ||| D

Solution |||
Normal Lapse rate also called Environmental Lapse Rate is the decrease in temperature with
height in the troposphere at an average rate of 6.4 0C per kilometer. However, normal lapse
rate is not constant as we go up in the atmosphere. It is affected by many factors such as
convection, radiation and condensation. In lower atmosphere, its average value is 6.50C in
lower atmosphere (troposphere).

76.Consider the following map: Which of the area(s) marked in the map above (with X, Z
and Y letters) represent arid and semi-arid climatic zones of India?

A. X only
B. Z only
C. X and Z only
D. None of the above

Answer ||| C

Solution |||

The areas marked with ‘X’ and ‘Z’ letters represent the hot ‘arid and semi arid’ climatic
zones respectively. The term “arid” normally means a region of the earth’s surface where
rainfall is nil or inadequate. Hence, the vegetation is nonexistent or sparse, agriculture
difficult or impossible and human living conditions precarious. Generally, arid regions are
those that receive less than 25 cm annual rainfall. It includes all hot and cold deserts. These
regions are devoid of any significant vegetation due to moisture stress. Regions receiving
annual rainfall between 25 and 75 cm can be termed as semi arid regions.

The hot Indian arid zone lies in the north-west part of the India and comprise largely of the
desert region of Rajasthan.
Semi-arid zones occur chiefly in the states of Rajasthan, Punjab, Haryana, Uttar Pradesh,
Gujarat, Madhya Pradesh, Maharashtra, Karnataka, Andhra Pradesh and Tamil Nadu as
shown in the map given above.

77.“Viticulture” is a common feature of which one of the following Australian cities?

A. Adelaide
B. Darwin
C. Hobart
D. Brisbane

Answer ||| A

Solution |||

Adelaide is famous for viticulture. Wine of Australia is very famous in the whole world. The
main producer of wine is southern state of Australia. Adelaide is the coastal city of South
America.

78.Which one of the following factors is NOT connected with planetary wind system?

A. Latitudinal variation of the atmospheric heating


B. Emergence of the pressure belt
C. Earths revolution around the Sun
D. Migration of the pressure belts due to apparent path of the Sun

Answer ||| C

Solution |||

The normal distribution of winds in the lower atmosphere is known as planetary winds.
Confined within some latitudinal belts, these winds blow and are basically controlled by the
latitudinal pressure belt. The main planetary winds are (i) north-east and southeast trade
winds (ii) temperate westerlies and (iii) polar easterlies, which fly from the polar high
pressure region to the temperate low east zone. The rotation of the Earth and its revolution
are not linked to the air system of the planets. The rotation of the Earth is responsible for the
generation of the Corollies force, which affects the direction of the planetary winds. That is
why we have southwest and north-east monsoon and no south and north monsoon. Once the
air is set in motion by the pressure gradient force, it passes through its path with an apparent
deflection, as seen by an observer on Earth.
79.Match List-I with List-II and select the correct answer using the code given below the
Lists:

A. A-4 B-2 C-1 D-3


B. A-4 B-1 C-2 D-3
C. A-3 B-1 C-2 D-4
D. A-3 B-2 C-1 D-4

Answer ||| B

Solution |||

A. Cyclones 1. India

B. Hurricanes 2. USA

C. Typhoons 3. East Asia

D. Willy-willies 4. Australia

80.Which one among the following substances is NOT a ‘green house’ gas?

A. Water vapour
B. Chlorofluorocarbon
C. Methane
D. Nitrogen

Answer ||| D

Solution |||

Greenhouse gas is a gas that absorbs and emits radiant energy within the thermal infrared
range. Greenhouse gases cause a greenhouse effect. The primary greenhouse gases in the
Earth's atmosphere are water vapor, carbon dioxide, methane, nitrous oxide, and ozone.

81.Which one among the following plains is associated with Limestone Topography?
A. Bajada plain
B. Alluvial plain
C. Karst plain
D. Pene plain

Answer ||| C

Solution |||

Karst is a topography formed by the dissolution of soluble rocks such as limestone. It is


underground drainage systems with sinkholes and caves characteristics. This has been
documented for more weathering resistant rocks such as quartzite, given the right conditions.

82.With reference to the soils found in India, consider the following pairs:

Soil Region

1) Alluvial : Indo-Gangetic Plains

2) Laterite Soils : Great Indian Desert

3) Black Soils : Deccan Trap

Which of the pairs given above is/are correctly matched?

A. 2 only
B. 1 and 2 only
C. 1 and 3 only
D. 1, 2 and 3

Answer ||| C

Solution |||

Alluvial Soils are the most common soils in India. These are widespread in northern plains
and river valleys. These are rich in humus, lime and organic matter. These are highly fertile.

The name of laterite soils is derived from Latin word ‘Later’ which means a Brick. These
soils are most common in the areas of high temperature and high rainfall and are formed due
to leaching. In India, laterite soils are found on the slopes of the Western Ghats and not in
Indian desert. Black Soils are most common in Deccan Trap. These are mature soils with
high water retaining capacity. These soils are ideal for cultivation of cotton.

83.Consider the following statement/s . Choose the correct option.


a) Cirrus clouds are formed at high altitudes with the height of 8,000 – 12,000m.

b) Cumulus clouds look like cotton wool.

c) Nimbus clouds are black or dark gray.

Codes-

A. A and B
B. B and C
C. A and C
D. All of these

Answer ||| D

Solution |||

* Cirrus Clouds- Cirrus clouds are formed at high altitudes (8,000 – 12,000m), these clouds
are the highest of all clouds and are composed entirely of ice crystals. These clouds
are precipitating clouds, although the ice crystals evaporate high above the earth’s
surface.They are thin and detached clouds having a feathery appearance. They are always
white in colour.

* Cumulus clouds are generally formed at a height of 4,000 -7,000 m, these clouds are look
like cotton wool. They exist in patches and can be seen scattered here and there and have a
flat base.They are Often called “fair-weather” clouds. cumulus clouds are common over
land on sunny days, when the sun heats the land creatingthermal convection currents.Each
thermal is distinct, and, consequently, each cumulus cloud is a distinct puff.

* Nimbus clouds are black or dark gray.They form at middle levels or very near to the
surface of the earth.These are extremely dense and opaque to the rays of the sun.Nimbus
clouds are shapeless masses of thick vapour.

84.‘The climate is characterised by an alternate hot, rainy season and cool, dry season. Trade
winds are prevalent here. This region in South America is also known as big game country as
people visit here for from across the world. Trees here are always present with luxuriant tall
grasses.’

Identify the region with the help of above sentence.

A. Equatorial Climatic Region


B. Sudan Climate
C. Temperate Grassland
D. Siberian Climate

Answer ||| B
Solution |||

The Savanna or Sudan Climate: (10o to 20oN & S)

* Climate is characterised by alternate hot, rainy season and cold, dry season.

* Trade winds are prevalent here.

* Vegetation is characterised by tall grass and short trees.

* They are also called Natural Zoo of the World.

* Elephant grasses are found here.

* Big game country in South America is known for people across the world visiting for
hunting.

* Important regions are- Llanos and Campos in South America, Kano and Salisbury in
Africa, etc.

85.Consider the following statements,

1) The intensity of the earthquake is measured by Mercalli Scale.

2) The magnitude of the Earthquake is measured by Richter scale.

3) Intensity is measured on the basis of the amount of energy released by an earthquake.

4) An earthquake has a single magnitude all over the area where it is felt.

Which of the statements given above is/are correct?

A. 1 only
B. 2 only
C. 2 and 3 only
D. 1, 2 and 4 only

Answer ||| D

Solution |||

Intensity is the rating of the effect of an earthquake at a particular place. It is based on the
observations of the damaged areas, using a descriptive scale like Mercalli Scale.

Magnitude is expressed with the help of Richter Scale and is measured on the basis of ground
motion recorded by an instrument seismograph.
Magnitude is measured as a logarithm of amount of energy released by an earthquake.

An earthquake has one magnitude. The magnitude does not depend on where the
measurement is made. On the other hand Intensity measures the amount of shaking at a
particular location. An earthquake causes many different intensities of shaking in different
areas.

86.Consider the following statements with reference to the Manjeera Wildlife Sanctuary:

1) It is located in Maharashtra state of India.

2) It has a Manjeera reservoir made on a tributary of the Godavari river.

Which of the statements given above is/are correct?

A. 1 only
B. 2 only
C. Both 1 and 2
D. Neither 1 nor 2

Answer ||| B

Solution |||

Manjeera wildlife sanctuary is located in Telangana State.

The Manjeera Wildlife Sanctuary has is a man-made reservoir, made on Manjeera river
which is a tributary of Godavari river. It also supplies drinking water to Hyderabad city. It is
home to vulnerable species of mugger crocodiles, and also provides lush nesting ground for
migratory birds. The Manjeera barrage and the Singur reservoir going dry this summer, has
forced the crocodiles from the Manjeera Wildlife sanctuary to search for any place where
water is logged.

87.Which of the following statements is/are correct with reference to ‘Biosphere Reserve
(BR)’?

1) The international designation of biosphere reserve is recognized by United Nation


Environment Programme (UNEP).

2) Biosphere Reserve’s main functions include both conservation of ecosystem and local
level sustainable development.

3) The tourism activities are strictly prohibited in a biosphere reserve.

Select the correct answer using the code given below:


A. 1 and 2 only
B. 2 only
C. 2 and 3 only
D. 1, 2 and 3

Answer ||| B

Solution |||

Biosphere Reserve (BR) is an international designation given by UNESCO (United Nations


Educational, Scientific and Cultural Organization) for representative parts of natural and
cultural landscapes over large area of terrestrial or coastal/marine ecosystems. BRs are
nominated by national government which meet a minimal set of criteria and a set of
conditions for inclusion in the world network of Biosphere reserves under the Man and
Biosphere Reserve Programme (MAB) of UNESCO.

Tripartite functions of BR (Conservation, Development and logistic support):

* Conserving biodiversity and integrity of plants and animal in their natural ecosystem.

* Sustainable use of natural resources by using technology for well-being of people.

* To provide areas for multi-faceted research and monitoring

* To provide facilities for education and training these objectives should be oriented in such a
way that the BRs are the units wherein the Biological, socio-economic and cultural
dimensions of conservation are integrated together into a, realistic sustainable conservation
strategy.

Biosphere reserves have three interrelated zones:

* Protected ecosystem is in core contributing in conservation of landscape, ecosystem,


species and genetic variations.

* Buffer zone surrounds the adjoining core areas for ecological practices, scientific research,
monitoring, training and education.

* Fostering, economic and human development that is socio-culturally and ecologically


sustainable is allowed in transition area.

Sustainable tourism like eco-tourism is allowed in transition zone of biosphere reserve to


promote livelihood, recreation and educational experience for the local people and students.

88.Which of the following is called the ‘Gateway to the Pacific’?

A. Suez Canal
B. Panama Canal
C. Bering Sea
D. Gulf of Alaska

Answer ||| B

Solution |||

The Panama Canal is known as the entrance to the Pacific, connects the Pacific Ocean (by the
Caribbean Sea) and the Atlantic Ocean. This canal is made by cutting the Isthmus of Panama
and is one of the main waterways in international trade. The Panama Canal currently governs
the United States.

89.Consider the following

1) Iron

2) Magnesium

3) Aluminium

4) Calcium

Which of the following is the correct sequence of quantity used in earth composition?

A. 1-2-3-4
B. 4-2-1-3
C. 1-4-2-3
D. 3-1-4-2

Answer ||| D

Solution |||

Major Elements in the Earth's Crust


90.The vertical movement of air is termed as:-

A. Wind
B. Air Current
C. Air Turbulence
D. Air Mobility

Answer ||| B

Solution |||

• The vertical movement of air is termed as air current.

• Wind is the horizontal movement of air.

• Turbulence is an irregular motion of the air resulting from eddies and vertical currents.

91.Which of the following is called the Red Planet: -

A. Pluto
B. Venus
C. Mars
D. Jupiter

Answer ||| C

Solution |||

• Mars is called for called the red planet, because the surface of Mars is red orange colour
because its soil has iron oxides and rust particles contains.

92.Which of the following is a tribe inhabiting the Andamans and Nicrobar Islands:-

A. Kukis
B. Santhals
C. Jarawas
D. Gonds

Answer ||| C

Solution |||
• The jarawas are an indigenous people of the Andaman island in India. The live in the parts
of South Andaman and middle Andaman island. The jarawas are recognised as an Adivasi
Andamanese Peoples.

93.Consider the following statements:

1) Areas where rainfall exceeds 250cm

2) Annual temperature 25°C - 27°C

3) Average humidity exceeds 75%

4) Trees do not shed the leaves

To which one of the following types of vegetation does the above represent?

A. Tropical wet evergreen


B. Tropical semi evergreen
C. Tropical moist deciduous
D. Tropical dry evergreen

Answer ||| A

Solution |||

Tropical wet evergreen vegetation mainly found in areas where annual rainfall occurs more
than 200cm and temperature is between 15 to 30℃, and due to the high rainfall average
humidity exceeds 75%. And in these areas trees also shed the leaves.

94.The longest river in the world is?

A. Nile
B. Brahmaputra
C. Amazon
D. Mississippi

Answer ||| A

Solution |||

* Nile is a major North flowing river in North Eastern Africa, is the longest river in Africa
and the longest river in the world (Brazilian government claims that the Amazon river is
longer than the Nile). The Nile is about 6650 kilometres long.
95.Which of the following is the largest producer of coffee in the world?

A. Argentina
B. Brazil
C. India
D. Sri Lanka

Answer ||| B

Solution |||

* Brazil is the largest Coffee producing country in the world and history comes from the last
150 years Vietnam is the second largest producer of coffee after Brazil with Total production
of about 1300000 ton per year.

96.Earth’s crust below the oceans is composed of?

A. Sedimentary rocks
B. Igneous rocks
C. Metamorphic rocks
D. Both b and c

Answer ||| A

Solution |||

* Sedimentary rocks are types of rocks that are formed by accumulation or deposition of
small particles and subsequent Cementation of mineral or organic particles on the floor of
ocean or other bodies of water at the Earth's surface. sedimentation may also offer as
dissolved minerals precipitate from water solution.

97.Consider the following statements:


1) Venus is the brightest planet in the night sky
2) Saturn is the least dense among all the planets
3) Mercury is the smallest and hottest planet of our solar system
Choose the correct option:

A. 1 and 2 only
B. 1 and 3 only
C. 2 and 3 only
D. 1, 2 and 3
Answer ||| A

Solution ||| • Venus is the brightest planet in the night sky. Therefore it is often called a
morning or an evening star, although it is not a star.
• Saturn is the least dense among all the planets. Its density is less than that of water.
• Mercury is the smallest and nearest planet to sun. However, it is not the hottest. It is because
mercury lacks environment so there is no gas to hold the sun’s radiation. Venus is the hottest
planet in solar system.

98.Which of the following statements are correct?


1) Isohyet is a line joining points of equal rainfall on a map in a given period.
2) Isohel is a line of equal or constant solar radiation.
3) Isobar is a line connecting places of same atmospheric pressure
4) Isohume is a line indicating equal cloud cover.
Select the correct answer from the codes given below:

A. 1, 2 and 3 only
B. 2, 3 and 4 only
C. 1, 3 and 4 only
D. 1, 2, 3 and 4

Answer ||| A

Solution ||| An isohume is a line of constant relative humidity. Equal cloud cover is indicated
by Isoneph.

99.There are two friends X & Y in two different countries of world. If at a time, Mr. X is in
France and his watch shows 2:00 PM and the watch of Mr. Y at the same time shows
4:40PM. Mr. Y can be in:

A. Algeria
B. Ghana
C. Ethiopia
D. Togo

Answer ||| C

Solution ||| France, Algeria, Ghana and Togo lie on Prime Meridian.
Since Mr. Y is ahead of Mr. X in time so Mr. Y should be in east of Mr. X.
The time difference is 160 minutes. 1 degree of longitude is equal to 4 minutes time
difference. Hence Mr. Y is 40 degree east of Mr. X.
Ghana, Algeria and Togo can’t be the answer as they all lie on prime meridian (students must
remember countries on prime meridian, tropic of cancer and Capricorn). Students should
know Algeria, Ghana and Togo lie in western side of Africa while Ethiopia is on the east.
The meridian of 40 degree east passes through Ethiopia. Hence Ethiopia is the answer.

100.Equinox is a time when:


1) the whole earth experiences equal days and equal nights
2) neither of the poles is tilted towards the sun
3) the plane of Earth's equator passes through the center of the Sun
Choose the correct option:

A. 1 and 2 only
B. 2 and 3 only
C. 1 and 3 only
D. 1, 2 and 3

Answer ||| D

Solution ||| On March 21 and September 23, the sun's direct rays fall on the equator. In this
situation, neither the pole is tilted towards the sun; Therefore, the whole earth experiences
similar days and equal nights. It is called the equinox.

101.With respect to weathering, consider the following statements:

1) Weathering accelerates erosion

2) Weathering helps in the enrichment and concentrations of certain valuable ores of iron

3) Weathering helps in the formation of soils

Choose the correct option:

A. 1 and 2 only
B. 2 and 3 only
C. 1 and 3 only
D. 1, 2 and 3

Answer ||| D

Solution |||

* The weathering process is responsible for breaking rocks into smaller pieces. Erosion
cannot be important if there are no rocks.
* By the removal of rocks and deposits, some of the valuable ores of iron, manganese,
aluminum, copper, etc. are helped in the enrichment and concentration, which are of great
importance to the national economy.

* Weathering is an important process in the formation of soils.

102.Dry climates are found in:

1) Area of subtropical high

2) On the western margin of the continents, adjoining the cold current

3) Interior of continents in mid latitudes

Choose the correct option:

A. 1 and 2 only
B. 2 and 3 only
C. 1 and 3 only
D. 1, 2 and 3

Answer ||| D

Solution |||

* Dry climates are characterised by very low rainfall that is not adequate for the growth of
plants.

* These climates cover a very large area of the planet extending over large latitudes from 15°
- 60° north and south of the equator.

* At low latitudes, from 15° - 30°, they occur in the area of subtropical high where
subsidence and inversion of temperature do not produce rainfall.

* On the western margin of the continents, adjoining the cold current, particularly over the
west coast of South America, they extend more equatorwards and occur on the coast land.

* In middle latitudes, from 35° - 60° north and south of equator, they are confined to the
interior of continents where maritime-humid winds do not reach and to areas often
surrounded by mountains.

103.Earth Day is celebrated annually on April 22 to demonstrate support for environmental


protection. Consider the following statements in this regard:
1) The theme of Earth Day 2019 is ‘Protect our Planet’
2) The Earth Day is being celebrated since first earth summit, 1992
Choose the correct option:

A. 1 only
B. 2 only
C. 1 and 2
D. Neither of the two

Answer ||| D

Solution ||| • The theme of Earth Day 2019 is ‘Protect our Species’.
• The earth day is being celebrated since 1970.
• World Earth Day is celebrated so that each of us remembers that we are all responsible for
the sustenance of Earth and its ecosystems.
• The day also identifies the collective responsibility to promote harmony with nature and the
Earth in order to achieve a balance among the economic, social and environmental needs of
present and future generations of humanity.

104.Consider the following statements:

1) Rainfall reduces from equator to poles

2) In mid latitudes (35-40 degrees), the rain is heavier on the western coasts as compared to
eastern coasts.

3) In higher latitudes (45-65 degrees), the rain is heavier on the eastern coasts as compared to
western coasts.

Choose the correct options:

A. 1 only
B. 2 and 3 only
C. 1, 2 and 3
D. None of the above

Answer ||| A

Solution |||

• Rainfall is maximum at equator due to extensive heating and evaporation of water.

• In mid latitudes (35-40 degrees), due to the easterlies, the rain is heavier on the eastern
coasts and goes on decreasing towards the west.

• In higher latitudes (45-65 degrees), due to the westerlies, the rainfall is first received on the
western margins of the continents and it goes on decreasing towards the east.
105.Which of the following affects the Indian monsoon:

1) The movement of the westerly jet stream

2) The Tibetan plateau

3) The shift of the position of Inter Tropical Convergence Zone (ITCZ) in summer

Choose the correct options:

A. 1 and 2 only
B. 2 and 3 only
C. 1 and 3 only
D. 1, 2 and 3

Answer ||| D

Solution |||

The following factors affect the Indian monsoon:

(A. The differential heating and cooling of land and water creates low pressure on the
landmass of India while the seas around experience comparatively high pressure.

(B. The shift of the position of Inter Tropical Convergence Zone (ITCZ) in summer, over the
Ganga plain (this is the equatorial trough normally positioned about 5°N of the equator. It is
also known as the monsoon-trough during the monsoon season).

(C.The presence of the high-pressure area, east of Madagascar, approximately at 20°S over
the Indian Ocean. The intensity and position of this high-pressure area affects the Indian
Monsoon.

(D. The Tibetan plateau gets intensely heated during summer, which results in strong vertical
air currents and the formation of low pressure over the plateau at about 9 km above sea level.

(E. The movement of the westerly jet stream to the north of the Himalayas and the presence
of the tropical easterly jet stream over the Indian peninsula during summer.

106.Which one of the following is not related to wind erosion?

A. Wind gap
B. Zeugen
C. Dreikanter
D. Demoiselle
Answer ||| A

Solution ||| Demoiselles


These are rock pillars which stand as resistant rocks above soft rocks as a result of differential
erosion of hard and soft rocks.
Zeugen
A table-shaped area of rock found in arid and semi-arid areas formed when more resistant
rock is reduced at a slower rate than softer rocks around it.
A Dreikanter
It is a type of ventifact that typically forms in desert or periglacial environments due to the
abrasive action of blowing sand.
A wind gap is a gap through which a waterway once flowed that is now dry as a result of
stream capture.

107.Which one of the following is the correct ascending sequence of States with regard to
percentage of urban population (2011)?

A. Tamil Nadu- Mizoram- Goa- Maharashtra


B. Goa- Mizoram- Maharashtra- Kerala
C. Maharashtra- Kerala- Mizoram- Goa
D. Mizoram- Goa- Maharashtra-Kerala

Answer ||| C

Solution ||| Percentage of urban population in Maharashtra (45.23), Kerala (47.72%),


Mizoram (51.51%) and in Goa (62.17%).

108.Which one of the following statements with regard to Jet stream, an upper level
tropospheric wave, is not correct?

A. It is narrow band of high-velocity wind.


B. It follows the wave path near the tropopause at elevations of 12 KM or Above 12 KM.
C. Jet streams are typically continuous over long distances.
D. In summer, the polar front jet achieves its maximum force.

Answer ||| B

Solution ||| the jet stream is more effective above the 12 km of the height of the surface of the
earth and the speed is 120 km/hr or more than it, the jet stream is found in the upper region of
troposphere.

109.Which among the following Plateaus is known as the Ruhr of India?


A. Chottanagpur Plateau
B. Malwa Plateau
C. Bundelkhand Plateau
D. None of the above

Answer ||| A

Solution |||

The Chottanagpur Plateau is known as the Ruhr of India.

Chottanagpur Plateau:

o It is the northeast part of the Peninsular plateau.

o It Involves Jharkhand, parts of the Chhattisgarh and West Bengal.

o This plateau comprises of series of step such as sub-plateaus (locally known as patlands –
high-level plateau). It is therefore famous as the Patland plateau and called as the Ruhr of
India.

o Rajmahal Hills are the north-eastern projection of the Chhota Nagpur Plateau.

o It is also a mineral-rich plateau.

Malwa Plateau:

o It lies in the Madhya Pradesh between the Aravali and Vindhyas.

o It comprises of the extensive lava flow and it is covered with the black soils.

o The extension of it is the Bundelkhand, the Bhaghelkhand & the Chota Nagpur Plateau

Bundelkhand Plateau:

o It lies along the borders of the UP and MP.

o Because of its intensive erosion, its semi-arid climate and the undulating area, it is unfit for
the cultivation.

o It has been transformed into the ravines by the extensive erosional activities of the river
Chambal and its tributaries.

110.Which of the following is a metamorphic rock?

A. Granite
B. Basalt
C. Marble
D. Pumice

Answer ||| C

Solution |||

Metamorphic Rocks are formed when rock changes over a duration of time due to physical
changes such as pressure, and heat. They can be either foliated or non-foliated.

They are also classified on the basis of the complexity of chemical molecules into ultramafic
rocks, carbonate rocks, pelites, marls, mafic, quartz, and feldspar. For example - marble,
granite, and gneiss.

Igneous Rocks are the rocks formed when molten rock cools to a solid-state. They can easily
be identified with their density, texture, colour, and mineral composition. For example -
Granite, Basalt and Pumice.

111.Which of the following is an east coast port?

A. V. O. Chidambaranar Port
B. Kamarajar Port
C. Nhava Sheva Port
D. Mormugao

Answer ||| B

Solution |||
112.Consider the following statement about “Prairies”-

1. They are Temperate grassland

2. They are found in Southern Part of African Continent

3. They are called the granaries of the world

Which of the above statement are correct?

A. 1 only
B. 1 and 3 only
C. 2 and 3 only
D. 1, 2 and 3

Answer ||| B

Solution |||

Steppe Climatic Region is also known as Temperate Grasslands. These are regions with
extremes of temperature where the summers are hot and the winters are cold and average
rainfall is about 50 cm.
Regional names of Temperate Grasslands are:-

* Canterbury - New Zealand

* Downs - Southern Australia

* Pustaz - Hungary and surrounding regions

* Prairies - North America

* Pampas - Argentina and Uruguay

* Veld - South Africa

Temperate Grasslands are characterized by the nearly treeless landscape and short grasses.
Due to such landscape, it is easier to use large mechanized appliances over large tracts and
this is facilitated by a moderate rainfall and the fertile soil which help to make the Prairies,
largest producers of the wheat in the world. This is the reason that Prairies are known as the
“Granaries of the world”. Major crops of Prairies are Wheat and maize.

113.Which of the following tree is not found in the deciduous forest of Northern India?

A. Deodar
B. Shisham
C. Peepal
D. Neem

Answer ||| A

Solution |||

Tropical Deciduous Forest also known as Monsoon Fores. They are a major Forest type
spread over a region which receives rainfall between 70 to 200 cm annually, this covers about
60% of the total geographical area of India.

Deciduous means “having a tendency to shed leaves annually/seasonally”. Once they shed
their leaves, it decomposes and enriches the soil thus making the soils in this biome fertile.

Based on the range of annual rainfall, the tropical deciduous forest is further divided into-

* Tropical Dry Deciduous (rainfall between 70-100cm )

* Tropical Moist Deciduous (rainfall between 100-200cm )

Tropical Dry Deciduous Forest-

* Found in areas receiving rainfall between 70 -100 cms.


* Largely found in parts of Eastern Rajasthan, Kathiawar, Central India, Central Deccan and
Punjab.

* Trees of such forest are usually less than 25 m high, with a light canopy.

* The trees shed their leaves completely with the onset of the hot dry season during March
and May.

* Important Trees associated with such forest are Teak, Laurel, Sal, Laurel, Sandalwood etc

Tropical Moist Deciduous Forest-

* Found in areas receiving rainfall between 100-200 cm and the mean annual temperature of
26°C.

* Largely found in Parts of Himalayan foothills, Maharashtra, eastern coastal plains, Andhra
Pradesh.

* These types of forests have dense trees and usually have 2 to 3 layer.

* Important Trees associated with such forest are Shishum, Peepal, Neem Tendu, Amla etc

114.Which of the following pair of port and location is correctly matched:

A. V. O. Chidambaranar Port -- Kochi


B. Nhava Sheva Port -- Mumbai
C. Kamarajar Port -- Ennore
D. Mormugao -- Goa

Answer ||| A

Solution |||
115.Which of the following landforms are formed by river streams during their youth stage?

1) V-Shaped valleys

2) Braided channel

3) Rapids

4) Oxbow lakes

Select the correct answer using the code given below.

A. 1, 2 and 3 only
B. 1 and 3 only
C. 1, 3 and 4 only
D. 1, 2, 3 and 4

Answer ||| B

Solution |||

A river stream forms a series of landforms on its course. The formation of landforms by a
river stream can be divided into three phases :

Youth

• Streams are few during this stage with poor integration and flow over original slopes
showing shallow V-shaped valleys with no floodplains or with very narrow floodplains along
trunk streams. Streams divides are broad and flat with marshes, swamp and lakes.

• Waterfalls and rapids may exist where local hard rock bodies are exposed.

Mature

• During this stage, streams are plenty with good integration. The valleys are still V-shaped
but deep; trunk streams are broad enough to have wider floodplains within which streams
may flow in meanders confined within the valley. The flat and broad inter stream areas and
swamps and marshes of youth disappear, and the stream divides turn sharp.

• Waterfalls and rapids disappear.

Old

• Smaller tributaries during old age are few with gentle gradients. Streams meander freely
over vast floodplains showing natural levees, oxbow lakes, Braided channels etc. Divides are
broad and flat with lakes, swamps and marshes. Most of the landscape is at or slightly above
sea level.

Hence, option (B) is the correct answer.


116.The Headquarters of Eastern Railway Zone of Indian Railway is located at

A. Bhubaneswar
B. Kolkata
C. Hajipur
D. Guwahati

Answer ||| B

Solution |||

The headquarters of the Eastern Railway Zone of Indian Railways is located at Kolkata. It
comprises

Of four divisions- Howrah, Malda, Sealdah and Asansol.


117.Which one of the following pairs of a river and its tributary is not correctly matched?

A. Godavari : Wainganga
B. Cauvery : Bhavani
C. Narmada : Amaravati
D. Krishna : Bhima

Answer ||| C

Solution |||

Amravati is a tributary of the Kaveri River. It starts at the Kerala / Tamil Nadu

border at the bottom of the Manjhampatti valley between Annamalai hills and Palni hills. It is
one of the longest rivers (282 km) in Tamil Nadu, which joins the Kaveri River near Karur.

118.Which one of the following describes the Lithosphere?

A. Upper and lower mantle


B. Crust and upper mantle
C. Crust and core
D. Mantle and core
Answer ||| B

Solution |||

Lithosphere is the rigid and outer most shell of the planet. It composed crust and upper
mantle. It behaves elastically on the time scale of a thousand years and more. It is sub-divided
into tectonic plates.

119.Which one of the following statements is not correct?

A. Kerala is the largest producer of natural rubber in India


B. Neyveli is an important thermal power generating area ofTamil Nadu
C. Ratnagiri Bauxite mining area is located in Kamataka
D. Assam is the largest tea producing state in India

Answer ||| C

Solution |||

Today, India is the world's third largest producer and fourth largest consumer of Natural
Rubber.

Kerala accounts for more than 90 percent of the country's total rubber production. The total
area under rubber cultivation in the state is 5.45 lakh hectares. The remaining natural rubber
is produced in Tamil Nadu and North East states.

Neyveli Thermal Power Station is a cluster of power plants located near lignite mines in
Neyveli, Tamil Nadu.

Ratnagiri bauxite mines are located in Maharashtra. Mostly, bauxite is mined in Odisha,
Jharkhand, Chhattisgarh and MP. Bauxite is a mineral found mostly in a belt around the
equator.

India is one of the largest producers of tea in the world, second only to China. About 52% of
the total tea produced in India comes from the state of Assam. The state recorded 652.95
million kg of tea production in the year 2015-16. Lakhimpur, Kamrup, Goalpara, Sivasagar,
Cachar, Nagaon and Derang districts of the state are the major producers of tea.

120.Movements of tides are mostly determined by –

A. Albedo effect
B. Wind velocity
C. Rotation of the Earth
D. Revolution of the Earth
Answer ||| C

Solution |||

Tides refer to the rise and fall of the surfaces of our oceans. This is due to the attractive forces
of the gravitational fields of the Moon and the Sun, as well as the centrifugal force caused by
the Earth's spin. As the positions of these celestial bodies change, so do the heights of the
surfaces

121.In India, the maximum amount of rainfall is received from -

A. Western Disturbances
B. North-East Monsoon
C. South-West Monsoon
D. Retreating Monsoon

Answer ||| C

Solution |||

The average annual rainfall is 300–650 millimetres, but much more unreliable than the rest of
India, with the southwest monsoon accounting for most of the rainfall. India receives
southwest monsoon winds in summer and northeast monsoon winds in winter. The southwest
monsoon is caused by the intense low-pressure system formed over the Tibetan Plateau. The
Northeast Monsoon is associated with high-pressure cells over the Tibetan and Siberian
plateaus.

122.Veliconda group of low hills is a structural part of

A. Nilgiri Hills
B. Western Ghats
C. Eastern Ghats
D. Cardamom Hills

Answer ||| C

Solution |||

Eastern Ghats comprise of a series of discontinuous low lying hills which include - Velikonda
hills, Odisha hills (Maliya hills), Nallamala hills, Palakonda hills, Javadi hills, and Shevaroy
hills.

Velikonda Ranges lie in Andhra Pradesh.


123.The most abundant gas in Erath’s atmosphere is:

A. Nitrogen

B. Oxygen

C. Carbon dioxide

D. Hydrogen

Answer ||| A

Solution |||

The atmosphere contains many gases, most in small amounts, including some pollutants and
greenhouse gases.

The most abundant gas in the atmosphere is Nitrogen and Oxygen is the second abundant gas
in the atmosphere. Argon, an inert gas is the third most abundant gas in the atmosphere.

124.Who among the following explains the concept of Biodiversity ‘Hotspots’:

A. Norman Myers

B. John Muir

C. G Nelson

D. None of the above

Answer ||| A

Solution |||

Norman Myers explains the concept of Biodiversity Hotspots.

Norman Myers CMG (born 24 August 1934 – 20 October 2019) was a British
environmentalist specializing in biodiversity and also noted for his work on environmental
refugees.

125.The Tropic of Cancer does not pass through which of the following Indian state-
A. Madhya Pradesh
B. West Bengal
C. Rajasthan
D. Odisha

Answer ||| D

Solution |||

Tropic of Cancer does not pass through Odisha.

Tropic of Cancer passes through 8 States i.e. Rajasthan, Gujarat, Madhya Pradesh,
Chhattisgarh, Jharkhand, West Bengal, Tripura and Mizoram.

Tropic of Cancer passes through a total of 16 countries across the world.

126.Collision-Coalescence process of precipitation is applicable to

A. clouds which extend beyond the freezing level


B. those clouds which do not extend beyond the freezing level
C. all types of clouds
D. cumulonimbus cloud

Answer ||| B

Solution |||

The Collision-Coalescence process is an important mechanism in forming raindrops in


warmer clouds (those with tops warmer than -15°C). In these warm clouds, raindrops form
exclusively by this process. Most tropical rain is formed in this way. The Collision-
Coalescence process is of relatively little importance in middle and high latitudes where most
precipitation begins high in the clouds where temperatures are well below freezing.

127.Which of the following is/ are the potential threat to the safety of large dams?

1) Urbanisation near dam sites

2) Flash floods in the catchment area

3) Seismic activities in the surrounding area

Select the correct answer using the code given below :


A. 1 only
B. 1 and 3 only
C. 2 and 3 only
D. 1, 2 and 3

Answer ||| C

Solution |||

A dam is a barrier across flowing water that obstructs or slows down the flow, often creating
a reservoir or lake. Dams are used for irrigation, electricity generation, controlling floods,
navigation etc. However, there are various threats to the large dams. Some of the important
treats are as follows:

• Sub-standard construction materials/techniques

• Geological instability caused by changes to water levels during filling or poor surveying

• Extreme inflow

• Flash floods in the catchment area

• Seismic activities in the surrounding area

• Sliding of a mountain into the reservoir

128.Which one of the following falls on the windward slope?

A. Mahabaleshwar
B. Pune
C. Bengaluru
D. Amritsar

Answer ||| A

Solution |||

Mahabaleshwar lies on the windward slope of the Western Ghats while Pune lies on the
leeward side. Bengaluru and Amritsar are both Continental locations.

Places on the Windward side receive very heavy rainfall as compared to places on the
Leeward side of a mountain range. This is due to moisture-bearing winds precipitating on the
windward side accounting for heavy rainfall on the windward side.
129.The headquarters of Southern Coast Railway is located in

A. Visakhapatnam
B. Chennai
C. Kolkata
D. Mumbai

Answer ||| A

Solution |||

The new railway zone Southern Coast Railway will be headquartered in Visakhapatnam.

130.A narrow strip of land that connects two larger bodies of land and is usually surrounded
by water is called?

A. Isthmus
B. Gulf
C. Strait
D. None of the Above

Answer ||| A

Solution |||
The country of Panama is the longest Isthmus, which measures 676 km in length and
connects North and south America.

A ‘Gulf’ is a large body of water that is surrounded by land. The world’s largest gulf is the
Gulf of Mexico.

A ‘Strait’ is a narrow strip of water, connecting two larger bodies of land or water.

131.Arrange the following tributaries of river Indus from North to South:

1) Chenab

2) Jhelum

3) Ravi

4) Sutlej

Select the correct answer using the code given below:

A. 4-3-1-2
B. 2-3-1-4
C. 1-2-3-4
D. 2-1-3-4

Answer ||| D

Solution |||

Jhelum, Chenab, Ravi and Sutlej are tributaries of Indus River. Jhelum is northernmost and
Satluj is southernmost river as shown in the figure below:
132.Tank irrigation is commonly found in South-Central parts of India. What could be the
reason?

1) Insufficient shallow ground water

2) Rocky plateau with impervious surface depression

3) Undulating terrain helps in the accumulation of rainwater in depression or man-made tank

Select the correct answer using the code given below:

A. 1 only
B. 1 and 2 only
C. 2 and 3 only
D. 1, 2 and 3

Answer ||| D

Solution |||

South-Central parts of India are made up of hard rock structure, so it is difficult to use tube
wells for irrigation. Further, digging of canals is also difficult and expensive on the rocky
surface of the peninsular plateau.
The Tank irrigation is practised in the rocky plateau area where the rainfall is uneven and
highly seasonal. Tank irrigation is commonly found in South-Central parts of India because
there is a little percolation of rain water due to hard rock structure and ground water is not
available in large quantities.

Further, the undulating terrain of south-central parts of India helps in the accumulation of
rainwater in depression or man-made tank.

133.Match List I with List II and select the correct answer using the code given below the
Lists:

List-I (Ocean current)

a) Humboldt

b) North Atlantic

c) Benguela

d) Agulhas

List-II (Coast)

1) Namibia – Angola

2) Chile – Peru Drift

3) Mozambique – Madagascar

4) United Kingdom – Norway

A. a-2 b-1 c-4 d-3


B. a-2 b-4 c-1 d-3
C. a-3 b-4 c-1 d-2
D. a-3 b-1 c-4 d-2

Answer ||| B

Solution |||

Humboldt current flows through Chile-Peru; North Atlantic Drift is found near United
Kingdom – Norway; Benguela current flows through Namibia – Angola and Agulhas current
flows across Mozambique – Madagascar.

134.Consider the following statements about the physiography of India:


1) The Deccan plateau receive heavy rainfall throughout the year as it falls on the wind ward
side of major hills of Central and Southern India.

2) The core of the great Himalayan is made up of limestone rocks.

3) The trough Narmada river is interposed between the Vindhyan and Satpura ranges.
Which of the statements given above is/are correct?

A. 1 and 2
B. 2 only
C. 2 and 3
D. 3 only

Answer ||| D

Solution |||

The Deccan plateau is a large triangular plateau, bounded by Vindhyan to the North and
flanked by the Eastern and Western Ghats.

Limestone can't bear the heavy weight of Himalayas. The core is actually made of Granite
Rocks.

Narmada River is one of the rivers in India that flows in a rift Valley, flowing West between
Satpura and Vindhyan ranges.

135.Consider the following

Statement 1: The two major plates were separated by the Tethys Sea and the Tibetan block
was close to the Asiatic landmass

Statement 2: the Height of the Himalayas is increasing even to this date.

Codes:

A. Both the statements are individually true and the statement 2 is the correct explanation of
statement 1.
B. Both the statements are individually true but statement 2 is not true explanation of
statement 1.
C. Statement 1 is true but statement 2 is false
D. Statement 1 is false but statement 2 is true

Answer ||| B

Solution |||
The two major plates were separated by the Tethys Sea and the Tibetan block was close to
the Asiatic landmass. During the Indian plate movement towards the Asiatic Plate, a major
event was the outbreak of lava and the formation of the Deccan Trap. It started around 60
million years ago and continued for a long time. The subcontinent was still close to the
Equator. 40 million years ago and thereafter, the formation of the Himalayas occurred.
Scientists believe that this process is still going on and the height of the Himalayas is
increasing even to this date.

136.Coromandal Coast is the?

A. Eastern Coast of India


B. Western Coast of India
C. Coast of Andaman Island
D. Coast of Nicobar Island

Answer ||| A

Solution |||

India has mainland coastline of 6100 Km. The Coromandel coast extends between Pulicat
Lake, Andhra Pradesh and Kanyakumari, Tamil Nadu.

• Konkan coast is Maharashtra coast + Goa coast;

• Malabar Coast is Kerala coast + Karnataka coast

• Coramandal coast is Tamil Nadu coast

• Utkal Coast is Odisha coast

137.Which among the following is the oldest Mountain range of India?

A. The Himalayas
B. Vindhyas
C. Aravalli
D. Eastern Ghat

Answer ||| C

Solution |||

The Aravallis located in the Northwestern India is one of the oldest ‘Fold Mountains’ of the
world. What we see now is residual mountains with an elevation of 300 meters to 900meters.
They stretch for a distance of 800 km from Himmat Nagar in Gujarat to Delhi.
Vindhyan Range are Non-Tectonic Mountains. They were formed by downward faulting of
the Narmada Rift Valley (NRV) to their south. They extend for 1200 km from Bharuch in
Gujarat to Sasaram in Bihar. They are younger than Aravallis and Satpura hills.

Eastern Ghats extend between the rivers Mahanadi and Vaigai. They comprise a series of
discontinuous hill ranges such as - Odisha hills (Maliya hills), Nallamala hills, Palakonda
hills, Velikonda hills, Javadi hills, and Shevaroy hills. Mahendragiri Peak (1501m) is the
highest point of Odisha hills.

138.The Appalachian Mountains are located in:

A. Africa
B. South America
C. North America
D. Europe

Answer ||| C

Solution |||

The Appalachian Mountains are located in eastern North America. Major important
mountains of world can be seen below:

139.Which of the following is not a Tropical Grassland?

A. Savanna
B. Llanos
C. Campos
D. Prairie
Answer ||| D

Solution |||

Grassland is an ecosystem in which the vegetation is dominated by grasses. Grassland


ecosystems are found in both Temperate regions and the Tropics. Tropical grasslands occur
on either side of the equator and extend till the tropics. This vegetation grows in the areas of
moderate to low amount of rainfall. The grass can grow very tall, about 3 to 4 metres in
height. Temperate grasslands are found in the mid latitudinal zones and in the interior part of
the continents. Usually, grass here is short and nutritious. Grasslands are known by different
names in different regions such as:

• Tropical Grasslands: East Africa- Savanna; Brazil- Campos; Venezuela- Llanos

• Temperate Grasslands: Argentina- Pampas; N. America- Prairie; S. Africa- Veld; C. Asia-


Steppe; Australia- Down

140.‘Namcha Barwa’ mountain peak is situated in ____________?

A. Arunachal Himalayas
B. Darjeeling and Sikkim Himalayas
C. Himachal and Uttarakhand Himalayas
D. Kashmir and Northwestern Himalayas

Answer ||| A

Solution |||

Namcha Barwa is a part of Arunachal Himalayas. It stands inside the Great Bend of the
Yarlung Tsangpo River. The river enters its notable gorge across the Himalaya.

141.Arrange the following rivers in increasing order to water availability.

1) Godavari

2) Kaveri

3) Ganga

4) Brahmaputra

5) Mahanadi

A. 2-5-1-3-4
B. 2-1-5-3-4
C. 1-2-3-5-4
D. 4-3-1-5-2

Answer ||| A

Solution |||

• According to Central Water Commission, water availability in following rivers in Million


Hectare Metre are:

1) Kaveri – 2.06

2) Mahanadi – 6.68

3) Godavari- 11.89

4) Ganga - 44.40

5) Brahmaputra – 59.07

142.Which one of the following is NOT true in reference to air mass?

A. Air mass forms either in tropical or in polar region


B. Air mass develops on continents as well as over ocean
C. Air mass develops in a cyclonic condition
D. Air mass changes the weather conditions

Answer ||| A

Solution |||

Air mass can be formed in tropical as well as polar regions. Air masses that form near the
equator or in tropical region are warmer than the one formed in polar regions also known as
Arctic Air Mass. Polar air mass is cold, dry and stable and generally originate over northern
Canada and Alaska as a result of radiational cooling.

143.The standard meridian of India is

A. 82 degrees 30’ E
B. 84 degrees 30’ E
C. 72 degrees 30’ E
D. 92 degrees 30’ E
Answer ||| A

Solution |||

• 82 degrees 30’ Eastern longitude defined as the standard meridian of India. 82 & 1/2 East
longitude is the standard meridian of India. It passes through Allahabad (Mirzapur) Uttar
Pradesh.

144.Which is the hottest planet of our solar system

A. Venus
B. Pluto
C. Mars
D. Mercury

Answer ||| A

Solution |||

• Venus is the hottest world in the solar system. Although Venus is not the planet closest to
sun, its dense atmosphere traps heat in a runaway version of the greenhouse effect that warms
Earth.

145.Choose the correct statements:


1) Comet is an icy and dusty object which revolves around the Sun in highly elliptical orbit
2) Asteroids are mainly made up of mineral and rock
3) Meteoroids are astronomical bodies composed of ice or rock and are larger than asteroids
Choose the correct option:

A. 1 and 2 only
B. 1 and 3 only
C. 2 and 3 only
D. 1, 2 and 3

Answer ||| A

Solution ||| • Asteroids are mainly composed of mineral and rock while the comets are
primarily composed of dust and ice. The period of revolution of comets round the Sun is
usually very long. A Comet appears generally as a bright head with a long tail. The length of
the tail grows in size as it approaches the sun. The tail of a comet is always directed away
from the sun
• Meteoroids can be composed of either ice/dust or minerals/rocks. Meteoroids are smaller
than asteroids.
146.With regard to Super-Earth, consider the following statements:
1) Super-Earth is a planet whose mass is higher than Earth
2) Super-Earth has somewhat similar habitability conditions as that of earth
3) Barnard's Star b is an example of Super-Earth
Choose the correct option:

A. 1 and 2 only
B. 1 and 3 only
C. 2 and 3 only
D. 1 only

Answer ||| B

Solution ||| • Super-Earth is a planet whose mass is higher than Earth but substantially lower
than the Uranus and Neptune.
• The term "super-Earth" refers only to the mass of the planet. It does not tell anything about
the habitability.
• In November, 2018, Astronomers discovered a frozen planet with a mass over three times
that of the Earth, orbiting the closest solitary star to the Sun. The name of this super-earth is
Barnard’s star b. This super-earth orbits Barnard's Star. Barnard’s star b is the second closest
known exoplanet to our Sun. The nearest light is more than four light years away from the
Earth.

147.Consider the following statements:

1) Stalagmite is a type of rock formation that rises from the floor of a cave.

2) Stalactite is a type of rock formation that hangs down from the ceiling of a cave.

3) Recently discovered Meghalayan is a Stalactite.

4) Studies of Stalagmite can help to predict droughts and floods.

Choose the correct option:

A. 1, 3 and 4 only
B. 1, 2 and 4 only
C. 1 and 2 only
D. 3 and 4 only

Answer ||| B

Solution |||
• Stalagmite is a type of rock formation that rises from the floor of a cave due to the
accumulation of material deposited on the floor from ceiling drippings. They form through
deposition of calcium carbonate and other minerals, which is precipitated from mineralized
water solutions. The corresponding formation hanging down from the ceiling of a cave is a
stalactite. The stalactites are formed by a constant supply of slow dripping water rich in
calcium carbonate (CaCO3) and carbon dioxide (CO2). They are found in limestone caves.

• Scientists have recently created a new phase in Earth’s geological history and named it
Meghalayan, after a stalagmite from a cave in the Indian state of Meghalaya that helped
define climatic events 4,200 years ago, marking the beginning of the phase that continues till
today. The Meghalayan Age began with a mega global drought that devastated ancient
agricultural civilisations from Egypt to ChinA. It is part of a longer period known as the
Holocene Epoch, which reflects everything that has happened over the past 11,700 years.

148.Which one of the following is the driest place in India

A. Drass

B. Leh

C. Jaisalmer

D. Shilong

Answer ||| A

Solution ||| Drass at Jasker river valley in Jammu and Kashmir is driest place in India.

149.Shivasamudram falls is located on which river?

A. Kaveri
B. Noyyal
C. Vaigai
D. Bhavani

Answer ||| A

Solution |||

Sivanasamudra is a city in the Mandya district of the state of Karnataka. It is situated on the
banks of the river Kaveri. The Shivanasamudra Falls, a segmented fall is on the Kaveri river.
150.If the latitude and longitudinal extent of an Indian State/UT is 15 °48’00’’ N to 14
°53’15’’ N and 74 °20’13’’ E to 74 °40’33’’ E, then which one of the following is that
State/UT?

A. Puducherry
B. Chandigarh
C. Goa
D. Delhi

Answer ||| C

Solution ||| Delhi: Latitude-28.38 N, Longitude-77.12 E


Chandigarh: Latitude-30.44 N, Longitude-76.47 E
Puducherry: Latitude- 11 ° 54' N, Longitude- 79 ° 48' E
Goa: Latitude- 15 °29'N. N, Longitude- 73 °48'E.

151.The jet streams are

A. wind systems with a pronounced seasonal reverse at a direction


B. winds blowing from the subtropic high-pressure belts towards the subpolar low-pressure
belts
C. narrow meandering bands of swift winds which blow in the tropopause and encircle the
globe
D. winds blowing from the sub-polar low-pressure belts towards the sudtropical high-
pressure belts

Answer ||| C

Solution ||| Jet streams are fast flowing, narrow, meandering, air currents found in the upper
atmosphere or in troposphere of some planets, including Earth. The main jet streams are
located near the altitude of the tropopause. The major jet streams on Earth are westerly
winds (flowing west to east).

152.Which is the correct arrangement of the following rivers from north to south?

A. Godavari, Penner, Kaveri, Periyar


B. Penner, Godavari, Periyar, Kaveri
C. Godavari, Kaveri, Penner, Periyar
D. Kaveri, Godavari, Periyar, Penner

Answer ||| A
Solution ||| The correct arrangement is Godavari, Penner, Kaveri, Periyar.
Hence Option A is correct

153.The National Water Academy (NWA) is located at

A. Dehradun
B. Hyderabad
C. Bhopal
D. Khadakwasla

Answer ||| D

Solution |||

The National Water Academy has the responsibility of streaming the water resources
personnel. It addresses the training needs of the water resource engineering of Central and
state agencies in terms of water resource projects. It includes planning, design, construction,
evaluation, operation, and monitoring. Its located at Khadakwasla.

154.Which among the following is the Right bank tributary of the Godavari river?

A. Penganga
B. Manjra
C. Indravati
D. Dharna

Answer ||| B

Solution |||

• The left bank tributaries are more in number and also larger in size than the right bank
tributaries. Left Bank Tributaries: Dharna, Penganga, Wardha, Wainganga, Pranahita
[conveying the combined waters of Penganga, the Wardha & Wainganga, Pench, Kanhan,
Sabari, Indravati etc.

• Right Bank Tributaries: Pravara, Manjra, Mula, Peddavagu, Maner etc. The Manjra (which
is of 724 km) is the only important right-bank tributary. It combines the Godavari after
passing through the Nizam Sagar.

• Below Rajahmundry, the river splits itself into two main streams, the Gautami Godavari on
the east side and the Vashishta Godavari on the west side and forms a large delta before it
pours into the Bay of Bengal. The Godavari delta is of lobate type with a round bulge and
several distributaries.
155.Which among the following statements is not correct regarding the tropical dry evergreen
forest?

A. They receive less than 100 cm of rainfall every year.


B. They are dense in nature.
C. They are found in many states such as Andaman and Nicobar Islands and Lakshadweep
islands.
D. They have many tree species such as Mahogany and Ebony.

Answer ||| A

Solution |||

Tropical Dry Evergreen Forests receive more than 200 cm of rainfall every year. They are
dense in nature. They are found in many states such as Andaman and Nicobar Islands and
Lakshadweep islands upper parts of Assam, Tamil Nadu, and western slopes of Western
Ghats.

They have many tree species such as Mahogany, Rosewood, Rubber, and Ebony. The trees in
these regions do not shed their leaves at the same time because there are no particular seasons
for shedding. It also has different types of vegetation, such as shrubs, trees, and creepers. The
animals found in this area are elephants, monkeys, and lemurs.

156.Match List I with List II and select the correct answer using the code
given below the Lists :

List I (Ultra mega power project at different stages of


development)

A) Sasan

B) Mundra

C) Tilaiya

D) Krishnapattam

List II (State)

1) Gujarat

2) Madhya Pradesh

3) Andhra Pradesh
4) Jharkhand

A. A-2 B-4 C-1 D-3


B. A-3 B-4 C-1 D-2
C. A-2 B-1 C-4 D-3
D. A-3 B-1 C-4 D-2

Answer ||| C

Solution |||

Status of Ultra Mega Power Projects (UMPPs) -

I. Sasan Ultra Mega Power Project in M.P- coal pithead

II. Mundra Ultra Mega Power Project in Gujarat- coastal

III. Krishnapatnam Ultra Mega Power Project in A.P.- coastal

IV. Ultra Mega Power Project in Jharkhand- coal pithead

V. Ultra Mega Power Project in Chhattisgarh- coal pithead

VI. Ultra Mega Power Project in Odisha - coal pithead

VII. Ultra Mega Power Project in Tamil Nadu ...coastal

VIII. Ultra Mega Power Project in Maharashtra- coastal

IX. Ultra Mega Power Project in Karnataka - coastal

You might also like